compendium omem - toomates

112
COMPENDIUM OMEM Olimpiada matemática de la Comunidad de Madrid (Fase local de la Olimpiada Matemática Española) 2007 2021 Gerard Romo Garrido

Upload: others

Post on 05-Oct-2021

13 views

Category:

Documents


0 download

TRANSCRIPT

Page 1: COMPENDIUM OMEM - Toomates

COMPENDIUM OMEM

Olimpiada matemática de la Comunidad de Madrid

(Fase local de la Olimpiada Matemática Española)

2007 – 2021

Gerard Romo Garrido

Page 2: COMPENDIUM OMEM - Toomates

Toomates Coolección

Los documentos de Toomates son materiales digitales y gratuitos. Son digitales porque están pensados para ser consultados

mediante un ordenador, tablet o móvil. Son gratuitos porque se ofrecen a la comunidad educativa sin coste alguno. Los libros de

texto pueden ser digitales o en papel, gratuitos o en venta, y ninguna de estas opciones es necesariamente mejor o peor que las otras. Es más: Suele suceder que los mejores docentes son los que piden a sus alumnos la compra de un libro de texto en papel, esto es un

hecho. Lo que no es aceptable, por inmoral y mezquino, es el modelo de las llamadas "licencias digitales" con las que las editoriales

pretenden cobrar a los estudiantes, una y otra vez, por acceder a los mismos contenidos (unos contenidos que, además, son de una bajísima calidad). Este modelo de negocio es miserable, pues impide el compartir un mismo libro, incluso entre dos hermanos,

pretende convertir a los estudiantes en un mercado cautivo, exige a los estudiantes y a las escuelas costosísimas líneas de Internet,

pretende pervertir el conocimiento, que es algo social, público, convirtiéndolo en un producto de propiedad privada, accesible solo a aquellos que se lo puedan permitir, y solo de una manera encapsulada, fragmentada, impidiendo el derecho del alumno de poseer

todo el libro, de acceder a todo el libro, de moverse libremente por todo el libro.

Nadie puede pretender ser neutral ante esto: Mirar para otro lado y aceptar el modelo de licencias digitales es admitir un mundo más injusto, es participar en la denegación del acceso al conocimiento a aquellos que no disponen de medios económicos, en un mundo

en el que las modernas tecnologías actuales permiten, por primera vez en la historia de la Humanidad, poder compartir el

conocimiento sin coste alguno, con algo tan simple como es un archivo "pdf". El conocimiento no es una mercancía. El proyecto Toomates tiene como objetivo la promoción y difusión entre el profesorado y el colectivo de estudiantes de unos

materiales didácticos libres, gratuitos y de calidad, que fuerce a las editoriales a competir ofreciendo alternativas de pago atractivas

aumentando la calidad de unos libros de texto que actualmente son muy mediocres, y no mediante retorcidas técnicas comerciales. Este documento se comparte bajo una licencia “Creative Commons”: Se permite, se promueve y se fomenta cualquier uso,

reproducción y edición de todos estos materiales siempre que sea sin ánimo de lucro y se cite su procedencia. Todos los documentos

se ofrecen en dos versiones: En formato “pdf” para una cómoda lectura y en el formato “doc” de MSWord para permitir y facilitar su edición y generar versiones parcial o totalmente modificadas. Se agradecerá cualquier observación, comentario o colaboración a

[email protected]

La biblioteca Toomates Coolección consta de los siguientes libros:

Bloques temáticos: Problem-solving Libros de texto (en catalán)

Geometría Axiomática pdf 1 2 ... 23

Problemas de Geometría pdf 1 2 3 4 5 6 7 8 9

Introducción a la Geometría pdf doc

Teoría de números

pdf 1 2 3

Trigonometría pdf doc pdf doc

Desigualdades pdf doc

Números complejos pdf doc pdf doc

Álgebra pdf doc pdf 1 2 3 4

Combinatoria

pdf doc

Probabilidad

pdf doc

Guía del estudiante de Olimpiadas Matemáticas

pdf

Combinatòria i Probabilitat pdf doc

Estadística pdf doc

Funcions pdf doc

Geometria analítica pdf 1 2

Àlgebra Lineal 2n batxillerat pdf doc

Geometria Lineal 2n batxillerat pdf doc

Càlcul Infinitesimal 2n batxillerat pdf 1 2

Programació Lineal 2n batxillerat pdf doc

Recopilaciones de pruebas PAU:

Catalunya TEC , Catalunya CCSS , Galicia , Portugal A , Portugal B

Recopilaciones de problemas olímpicos y preolímpicos (España):

OME , OMEFL , OMEC , OMEM , Canguro , Cangur

Recopilaciones de problemas olímpicos y preolímpicos (Internacional):

IMO , OMI , AIME , Kangourou , AMC 8 , AMC12 (2008-2020) , SMT

Versión de este documento: 02/03/2021

Todos estos documentos se actualizan constantemente. ¡No utilices una versión anticuada! Descarga totalmente gratis la última

versión de los documentos en los enlaces superiores.

www.toomates.net

Page 3: COMPENDIUM OMEM - Toomates

Índice.

Fase 0 Fase 1

Enunciados Soluciones Enunciados Soluciones

43 - XLIII - (2006-07) 4 8 10

44 - XLIV - (2007-08) 11 16

45 - XLV - (2008-09) 17 21

46 - XLVI - (2009-10) 22 26

47 - XLVII - (2010-11) 27 31

48 - XLVIII - (2011-12) 32 36

49 - XLIX - (2012-13) 39 43

50 - L - (2013-14) 49 53

51 - LI - (2014-15) 54 58 59 60

52 - LII - (2015-16) 62 66

53 - LIII - (2016-17) 67 71 72

54 - LIV - (2017-18) 73 77 79

55 - LV - (2018-19) 80 84

56 - LVI - (2019-20) 85 89 90 91

94 (V. personal)

57 - LVII - (2020-21) 100 104 106 (V. personal)

Fuentes.

https://www.ucm.es/sociedadpuigadam/todas-las-olimpiadas

Todo este material ha sido agrupado en un único archivo "pdf" mediante la aplicación

online

https://www.ilovepdf.com/

Page 4: COMPENDIUM OMEM - Toomates

REAL SOCIEDAD MATEMÁTICA ESPAÑOLA

XLIII OLIMPIADA MATEMÁTICA ESPAÑOLAComunidad de Madrid

Primera sesión, viernes 24 de noviembre de 2006

• En la hoja de respuestas, rodea con un círculo la opción que creas correcta en cada pregunta. Si decides cambiarla, táchala con una cruz y escoge otra.

• Cada respuesta correcta te aportará 5 puntos, cada respuesta en blanco 2 puntos y cada respuesta errónea, 0 puntos.

• No están permitidas calculadoras ni ningún instrumento de medida.

• Tiempo: 3 horas.

¿Cuántos subconjuntos de 3 elementos del conjunto {88, 95, 99, 132, 166, 173} tienen la propiedad de que la suma de sus elementos es un número par?

A) 6 B) 8 C) 10 D) 12 E) 24

Algunos enteros positivos tienen estas propiedades:I La suma de los cuadrados de sus cifras es 50.II Cada cifra es mayor que la que está a su izquierda.

¿Cuál es el producto de las cifras del mayor de ellos?

A) 7 B) 25 C) 36 D) 48 E) 60

Para enumerar las páginas de un libro, empezando por la página 1, hemos necesitados 600 dígitos (por ejemplo, para numerar la página 23 hemos necesitado 2 dígitos, y para numerar la 122 hemos necesitado 3). ¿Cuántas páginas tiene el libro?

A) 136 B) 137 C) 236 D) 600 E) Nada de lo anterior.

La igualdad FGEDCBA =++++ representa la suma de cinco números de una cifra que es igual a un número de dos cifras, donde todas las cifras que aparecen son distintas. Si el número FG es el mayor posible, ¿cuál es el valor de G ?

A) 1 B) 2 C) 3 D) 4 E) 5

En una hoja de papel hay escrito un número. De las cuatro afirmaciones siguientes, tres son verdaderas y la otra es falsa.

I El número es el 1. III El número no es el 3.II El número es el 2. IV El número no es el 4.

De los siguientes enunciados, ¿cuál de ellos es siempre correcto?

A) I es falsa B) II es verdadera C) II es falsa D) III es falsa E) IV es verdadera

1

2

3

4

5

Page 5: COMPENDIUM OMEM - Toomates

Las gráficas de baxy +−−= e dcxy +−= se cortan en los puntos (2, 5) y (8, 3). El valor de ca + es:

A) 5 B) 7 C) 8 D) 10 E) 13

El triángulo de lados 3, 4, y 5 divide el interior de su circunferencia circunscrita en cuatro regiones. Sean A, B y C las áreas de las regiones no triangulares, siendo C la mayor. Entonces:

A) CBA =+ B) 222 CBA =+ C) CBA =++ 6 D) CBA 534 =+ E) 222

111CBA

=+

Las letras a, b, c, d, e, f del dibujo adjunto han sido sustituidas por los números 2, 4, 5, 6, 8, 9 (en algún orden) de forma que la suma de los elementos de cada fila y cada columna es siempre el mismo número, k. ¿Qué número es k?

A) 15 B) 16 C) 17 D) 19 E) 21

Si x, y, z son números reales que satisfacen las igualdades 41 =+y

x , 11 =+z

y , 371 =+

xz ,

el valor de zyx es:

A) 32

B) 1 C) 34

D) 2 E) 37

Desde un determinado punto de la hipotenusa de un triángulo rectángulo, trazamos paralelas a los catetos de éste que dividen al triángulo en un cuadrado y dos triángulos rectángulos pequeños. Si el área de uno de estos triángulos pequeños es m veces el área del cuadrado, ¿cuál es el cociente entre el área del otro triángulo y el área del cuadrado?

A) 12

1+m

B) m C) m−1 D) m41

E) 281m

El segmento que une los puntos medios de las diagonales de un trapecio mide 3 cm de longitud. Si la base mayor del trapecio mide 97 cm, la longitud, en centímetros, de la base menor es:

A) 94 B) 92 C) 91 D) 90 E) 89

En la siguiente igualdad, cada letra representa una cifra (letras distintas, cifras distintas) y cada palabra un número de dos o tres cifras. ( ) ( ) MMMCAVA =· . ¿Cuánto vale

VMCA +++ ?

A) 19 B) 20 C) 21 D) 22 E) 24

Para cada número real x , sea )(xf el mínimo de los números 14 +x , 2+x y 42 +− x . ¿Cuál es el máximo valor para )(xf ?

A) 31

B) 21

C) 32

D) 25

E) 38

7 a b 1c d3 e f 10

6

7

8

9

10

11

12

13

Page 6: COMPENDIUM OMEM - Toomates

Sea f una función lineal tal que )2()1( ff ≤ y )4()3( ff ≥ . Si 5)5( =f , ¿cuál de las siguientes afirmaciones es verdadera?

A) 0)0( <f B) 0)0( =f C) )1()0()1( −<< fff D) 5)0( =f E) 5)0( >f

Si a, b y c son tres números (no necesariamente distintos) elegidos al azar en el conjunto {1, 2, 3, 4, 5}, la probabilidad de que cab + sea par es:

A) 52

B) 12559

C) 21

D) 12564

E) 53

Los puntos )9,3(A , )1,1(B , )3,5(C y ),( baD están en el primer cuadrante y son los vértices del cuadrilátero ABCD. Si el cuadrilátero formado uniendo los puntos medios de los lados AB, BC, CD y DA es un cuadrado, ¿cuál es la suma de las coordenadas del punto D?

A) 7 B) 9 C) 10 D) 12 E) 16

El producto de los cuatro enteros positivos a, b, c y d es 8!. Si 524=++ baab , 146=++ cbbc y 104=++ dccd , ¿cuánto vale da − ?

A) 4 B) 6 C) 8 D) 10 E) 12

Para cada entero positivo 1>m , representamos por P(m) el mayor factor primo de m. Por ejemplo 7)35( =P y 17)289( =P . ¿Para cuántos enteros positivos se verifica que

nnP =)( y 48)48( +=+ nnP ?

A) 0 B) 1 C) 3 D) 4 E) 5

En el triángulo ABC tenemos que 25=AB , 39=BC y 42=AC . Los puntos D y E, que están en AB y AC respectivamente, hacen que 19=AD y 14=AE . ¿Cuál es el cociente entre el área del triángulo ADE y el área del cuadrilátero BCDE?

A) 1521266

B) 7519

C) 31

D) 5619

E) 1

Todos los números de teléfonos móviles que nos han ofrecido para el comité organizador de la XLIII Olimpiada Matemática Española son de la forma 66abcdefg donde a, b, c, d, e, f y g son números distintos, no son ni 0 ni 1 y además están en orden creciente. ¿Cuántos números de teléfono nos han ofrecido?

A) 1 B) 2 C) 6 D) 7 E) 8

¿Cuál es la media de todos los números de 5 cifras distintas que pueden formarse con las cifras 1, 3, 5, 7 y 8?

A) 48.000 B) 49.999,5 C) 53.332,8 D) 55.555 E) 56.432,8

¿Para cuántos enteros positivos n menores o iguales que 24 se verifica que n! es divisible entre n+++ ...21 ?

A) 8 B) 12 C) 16 D) 17 E) 21

16

17

18

19

20

21

22

15

14

Page 7: COMPENDIUM OMEM - Toomates

Un subconjunto B del conjunto de los enteros 1, 2, …, 100 tiene la propiedad de que no tiene ningún par de elementos que sumen 125. ¿Cuál es el máximo número de elementos que puede tener el subconjunto B?

A) 50 B) 51 C) 62 D) 65 E) 68

En el conjunto {2, 22, 23, …, 225} elegimos dos números distintos a y b. ¿Cuál es la probabilidad de que balog sea un entero?

A) 252

B) 30031

C) 10013

D) 507

E) 21

Las dos raíces de la ecuación de segundo grado 0632 =+− kxx son números primos. ¿Cuántos valores puede tomar k?

A) 0 B) 1 C) 2 D) 4 E) Más de 4

Supón que a y b son dígitos distintos, ninguno cero ni nueve y que el número periódico ...,0 abababab viene expresado como fracción irreducible. ¿Cuántos denominadores

diferentes podría haber?

A) 3 B) 4 C) 5 D) 8 E) 9

En el triángulo ABC, el lado AC y la mediatriz del lado BC se cortan en el punto D, siendo BD bisectriz del ángulo B. Si 9=AD y 7=DC , el área del triángulo ABD es:

A) 14 B) 21 C) 28 D) 514 E) 528

En el triángulo rectángulo ABC de la figura, la hipotenusa AB mide 10 cm y el ángulo B, 60º. Elegimos al azar un punto P en el interior de dicho triángulo y prolongamos la recta BP hasta que corte en D al lado AC. ¿Cuál es la probabilidad de que 25>BD ?

A) 2

22 − B) 31

C) 3

33 − D) 21

E) 5

55 −

En el triángulo ABC, el lado AB mide 1 y el AC, 2. Si el otro lado, BC, y la mediana desde A son de igual longitud, ¿cuál es esta longitud?

A) 2

21 + B) 2

31 + C) 2 D) 23

E) 3

Para cada entero n mayor que 1, definimos 2006log1

nna = . Si 32 aab += y 59514 aaac ++= ,

cb − es igual a:

A) – 2 B) – 1 C) 2006

1D)

20031

E) 21

ADC

B

10P

23

24

25

26

27

28

29

30

Page 8: COMPENDIUM OMEM - Toomates

REAL SOCIEDAD MATEMÁTICA ESPAÑOLA

XLIII OLIMPIADA MATEMÁTICA ESPAÑOLAComunidad de Madrid

Primera sesión, viernes 24 de noviembre de 2006

Nombre: ……………………………………………..…………. Curso:……………..……..

IES/Colegio: …………………………………..…… Teléfono de contacto:…......…………

Pregunta Respuesta Pregunta Respuesta1 C 16 C2 C 17 D3 C 18 B4 B 19 D5 E 20 E6 D 21 C7 C 22 C8 D 23 C9 B 24 B10 D 25 B11 C 26 A 12 C 27 D13 E 28 C14 D 29 C15 B 30 B

Correctas Incorrectas En blanco TOTAL

Page 9: COMPENDIUM OMEM - Toomates
Page 10: COMPENDIUM OMEM - Toomates

Problemas propuestos en la Fase Local de la XLIII Olimpíada Matemática Española

Problema 1

Demostrar que es imposible obtener un cubo yuxtaponiendo tetraedros regulares, todos del mismo tamaño. Problema 2

Entre los 2007 primeros enteros positivos elegimos 1005 cualesquiera. Demostrar que seguro que hay al menos dos de estos 1005 cuya diferencia es 4. Problema 3

Demostrar que, en un triángulo, la distancia de un vértice cualquiera al ortocentro es el doble de la distancia del circuncentro al lado opuesto a ese vértice. Problema 4

Consideramos un triángulo isósceles ABC con AC = BC, y un punto D fuera del triángulo tal que el ángulo sea el doble del ángulo ACB∠ ADB∠ . La recta AD corta a BC en el punto E con CE = 2 y EB = 1. Calcular el producto AE·ED Problema 5

Encontrar todas las soluciones enteras posibles, x e y, de la ecuación:

( )p x y xy+ =

siendo p un cierto número primo. Problema 6

Sea la sucesión {2, 9, 28, 65, . . .} y 31na = + n ( )1mcd ,n na aδ += n Hallar el

máximo valor que puede tomar nδ .

Page 11: COMPENDIUM OMEM - Toomates

REAL SOCIEDAD MATEMÁTICA ESPAÑOLA

XLIV OLIMPIADA MATEMÁTICA ESPAÑOLA Comunidad de Madrid

Primera sesión, viernes 23 de noviembre de 2007

• En la hoja de respuestas, escribe la letra que corresponde a la opción que creas correcta

en cada pregunta. Si decides cambiarla, táchala con una cruz y escribe otra. • Cada respuesta correcta te aportará 5 puntos, cada respuesta en blanco 2 puntos y cada

respuesta errónea, 0 puntos. • No están permitidas calculadoras ni ningún instrumento de medida. • Tiempo: 3 horas.

A un número de 4 cifras del que sabemos que es divisible por 3, 4 y 5, se le han borrado las dos últimas cifras, quedándonos el 86. ¿Cuál es la suma de las dos cifras borradas?

A) 4 B) 6 C) 7 D) 9 E) 14 Utilizando algunos de los nueve enteros que hay del 11 al 19, podemos hacer una lista de forma que cualesquiera dos consecutivos no son primos entre sí. Por ejemplo: 16, 18, 15, 12, 14. Al hacer la lista más larga posible con los nueve enteros que hay del 111 al 119, ¿cuántos quedarán fuera?

A) 0 B) 1 C) 2 D) 3 E) 4 Al hacer la multiplicación de los dos números de dos cifras que ves, se nos han perdido unos cuantos. ¿Qué cifra debe aparecer en lugar de ♥?

A) 1 B) 3 C) 5 D) 7 E) 9

Los cuatro números 21 , x , y ,

43 están colocados en orden creciente. Si la diferencia entre

cada dos consecutivos es constante, ¿cuál es valor de y ?

A) 83 B)

32 C)

127 D)

65 E)

85

La suma de un número de tres cifras y la suma de sus cifras es 429. ¿Cuál es el producto de las tres cifras de este número?

A) 20 B) 28 C) 30 D) 36 E) 48

1

2

3

4

5

4 ~ × ~ ~ ~ 8 ~ 8 ~ 0 ~ ~ 4 ♥

Page 12: COMPENDIUM OMEM - Toomates

Colocamos cinco rectángulos idénticos como se muestra en la figura. Si el rectángulo grande mide 15 cm de ancho, ¿cuál es su área en cm2?

A) 270 B) 300 C) 330 D) 360 E) 450

¿Cuándo es el producto ⎟⎠⎞

⎜⎝⎛ +⎟

⎠⎞

⎜⎝⎛ +⋅⎟

⎠⎞

⎜⎝⎛ +⋅⎟

⎠⎞

⎜⎝⎛ +

n11

411

311

211 L un número entero?

A) Cuando n es impar B) Cuando n es par C) Cuando n es múltiplo de 3 D) Siempre E) Nunca Doblamos una hoja de papel de dimensiones 1 y

2 como se muestra en la figura, es decir, que una esquina cae en el lado opuesto y el doblez pasa por la otra esquina opuesta al lado. ¿Cuál es el valor de d ?

A) 21 B) 12 − C)

167 D) 23 − E)

32

Las siguientes escalas muestran la población de cinco ciudades A, B, C, D y E en 1997 y 2007. ¿Cuál de las cinco tiene un porcentaje de crecimiento más alto durante estos diez años?

A) A B) B C) C D) D E) E En la circunferencia de la figura, de radio 1 cm, AB es un diámetro. Hemos dibujado dos arcos del mismo radio, con centros A y B, que se cortan, como puedes observar, en puntos que están en la circunferencia. ¿Cuál es, en cm2, él área sombreada?

A) 2π B) 1 C) 1−π D) 2 E)

32π

El dibujo que ves muestra un paralelogramo dentro de un triángulo. Si los segmentos marcados tienen igual longitud, ¿cuántos grados mide el ángulo x ? A) 80º B) 85º C) 90º D) 95º E) 100º

6

7

8

9

10

11

15 cm

1

2

d 90º

0 50 100 150

0 50 100 150

A B C D E

A B C D E

Población en 1997

Población en 2007

x

B A

Page 13: COMPENDIUM OMEM - Toomates

¿Qué fracción del área del rectángulo ABCD está sombreada?

A) 8116 B)

94 C)

92

D) 81 E)

91

Si 111111110111

=x , 223222221222

=y , 334333331333

=z , ¿qué afirmación de las siguientes es correcta?

A) zyx << B) yzx << C) xzy << D) yxz << E) zxy <<

Si a personas trabajando b horas diarias cada una, pintan c m2 de pared, ¿cuántas horas debe trabajar al día cada una de d personas para pintar e m2?

A) cdabe B)

ceabd C)

deabc D)

beacd E)

bdace

EL dibujo muestra un rectángulo x×1 dentro de un cuadrado 1010 × . ¿Cuál es el valor de x? A) 2210 + B) 1210 − C) 2210 − D) 210 + E) 12

El cociente entre la longitud y la anchura de una pantalla de televisión de las de antes es 34 , y

ese cociente en las de pantalla plana es 9

16 . Si dos pantallas, una antigua y otra plana, tienen

la misma área, el cociente (longitud pantalla plana):(longitud pantalla antigua) es:

A) 3

2 B) 23 C)

23 D)

34 E) Nada de lo anterior

El dibujo que ves muestra un cuadrado y dos segmentos que unen vértices del cuadrado con el punto medio de un lado opuesto. Si la figura sombreada es un rectángulo, ¿qué fracción del área del cuadrado ocupa?

A) 31 B)

52 C)

103 D)

21 E)

83

12

13

16

17

15

14

8 1

A B

C D

x

Page 14: COMPENDIUM OMEM - Toomates

Un cuadrado está dividido en cuatro rectángulos iguales y una cuadrado pequeño como se muestra en

la figura. Si el área del cuadrado pequeño es 41 del

área del grande, ¿cuál es el cociente entre las longitudes de los lados de los rectángulos?

A) 2

1 B) 3

1 C) 21 D)

31 E)

41

En el rectángulo de la figura, hemos sombreado tres cuartos de su área. ¿Cuál es el valor de x?

A) 2 B) 2,4 C) 3

D) 3,6 E) 4

En el cuadrilátero ABCD de la figura, la diagonal

AC es bisectriz del ángulo A. Si 37

=AB y

421=AD , y los ángulos CBA ˆ y DCA ˆ son rectos,

la longitud de dicha diagonal es:

A) 27 B) 4 C)

417 D)

42193 E) 5

Si 012 =++ xx , ¿cuánto vale 22 1

xx + ?

A) i341

+ B) 1 C) i−− 3 D) 1− E) Nada de lo anterior

De los siguientes números hay uno sólo que es primo. ¿Cuál?

A) 22 1110001 + B) 22 666555 + C) 22 00420072 − D) 22 00620072 + E) 22 00920072 +

Si R es el radio de la circunferencia circunscrita a un decágono regular de lado 1, ¿cuántas de las siguientes expresiones son iguales a R ?

º36º18cos

sen;

253 + ; ( )º36cos12

1−⋅

; º542 sen⋅

A) 0 B) 1 C) 2 D) 3 E) 4

18

19

20

21

22

23

6

6

2 x

90º 90º

A

B C

D

Page 15: COMPENDIUM OMEM - Toomates

Hace 22 años, la edad de Jorge era el cuádruple de la de Marta, pero hace 18 años era sólo el triple. ¿Cuántos años hace que era nada más que el doble?

A) 14 B) 8 C) 7 D) 6 E) 5 En el rectángulo ABCD de centro O , el punto E está en el segmento OC y F es el pie de la perpendicular de E a

OB . Si 3=OF y 4=EF , el cociente BCAB es igual a:

A) 43 B) 1 C)

54 D)

34 E) 2

En el interior del triángulo equilátero ABC elegimos al azar un punto P. ¿Cuál es la probabilidad de que el ángulo APB sea obtuso?

A) 3

1 B) 21

6+

π C) 32

π D) 21

36+

π E) 43

En la sucesión 1, 3, 4, 7, 11, … se verifica, como ves, que cada término después de los dos primeros es la suma de los dos que le preceden. De las siguientes afirmaciones, ¿cuántas son verdaderas? I. El 20º término es divisible por 2. II. El 30º término es divisible por 3. III. El 40º término es divisible por 4. IV. El 50º término es divisible por 5.

A) 0 B) 1 C) 2 D) 3 E) 4

Si b es un número real que verifica 13 += bb , ¿cuál de las siguientes igualdades no es correcta?

A) bbb += 24 B) 145 += bb C) 1234 −+= bbb

D) 1

1112

−+=++

bbb E) 1234 +=+ bbb

En el triángulo isósceles ABC con ACAB = y el ángulo A igual a 2α , las alturas que parten

de A y B se cortan en el punto P. Si E es el pie de la altura que parte de A, ¿cuánto vale AEPE ?

A) 41 B)

31 C) αsen D)

αcos1 E) α2tg

Una bolsa contiene m bolas rojas y n bolas blancas. Elegimos al azar una bola, miramos su color y la devolvemos a la bolsa junto a k bolas de su mismo color. A continuación elegimos una segunda bola al azar. ¿Cuál es la probabilidad de que esta segunda bola sea roja?

A) nm

m+

B) nm

n+

C) knm

m++

D) knm

km++

+ E) knm

nm++

+

24

25

26

27

28

29

30

A

O E

F

C

B

D

Page 16: COMPENDIUM OMEM - Toomates

REAL SOCIEDAD MATEMÁTICA ESPAÑOLA

XLIII OLIMPIADA MATEMÁTICA ESPAÑOLA Comunidad de Madrid

Primera sesión, viernes 24 de noviembre de 2006

Nombre: ……………………………………………..…………. Curso:……………..…….. IES/Colegio: …………………………………..…… Teléfono de contacto:…......…………

Pregunta Respuesta Pregunta Respuesta 1 A 16 A 2 C 17 B 3 D 18 D 4 B 19 E 5 B 20 A 6 A 21 D 7 A 22 A 8 B 23 E 9 C 24 D

10 D 25 E 11 C 26 D 12 A 27 B 13 B 28 E 14 A 29 E 15 B 30 A

Correctas

Incorrectas En blanco TOTAL

Page 17: COMPENDIUM OMEM - Toomates

REAL SOCIEDAD MATEMÁTICA ESPAÑOLA

XLV OLIMPIADA MATEMÁTICA ESPAÑOLA Comunidad de Madrid

Primera sesión, viernes 28 de noviembre de 2008

• En la hoja de respuestas, escribe la letra que corresponde a la opción que creas correcta

en cada pregunta. Si decides cambiarla, táchala con una cruz y escribe otra. • Cada respuesta correcta te aportará 5 puntos, cada respuesta en blanco 2 puntos y cada

respuesta errónea, 0 puntos. • No están permitidas calculadoras ni ningún instrumento de medida. • Tiempo: 3 horas.

El valor de 44 32 + es:

A) 4 B) 20 C) 5 D) 7 E) 97 Si 25166 +=− yx y 2596 +=− xy , ¿cuál es el valor de yx − ?

A) 1 B) 21+ C) 2 D) 22− E) 0 El diagrama muestra dos cuadrados que se solapan. ¿Cuánto vale la suma de los ángulos x e y?

A) 180º B) 300º C) 330º D) 360º E) Falta información ¿Qué operación debe reemplazar a ▲ para que la igualdad escrita sea correcta?

1·2·(3▲4+5)·(6·7+8+9)+2=2008 A) + B) – C) · D) : E) Ninguna de las anteriores

Juanje calculó correctamente el valor de 58·85. ¿Cuántos dígitos tenía su respuesta?

A) 11 B) 12 C) 16 D) 14 E) 15

Si la media de los tres números x, y, z es x, ¿cuál es la media de los dos números y, z?

A) 2x B) x C) 2x D) 3x E) 4x

1

2

3

4

6

5

Page 18: COMPENDIUM OMEM - Toomates

Emparejamos los números 72, 8, 10, 5, 45, 36, 15 y 24 de forma que el producto de cada pareja sea el mismo. ¿Con qué número hemos emparejado 10? A) 36 B) 45 C) 24 D) 15 E) 72 ¿Cuál de las siguientes gráficas puede ser la que muestra la longitud de una circunferencia c en función de su diámetro d ? A) B) C) D) E)

El dibujo que ves muestra un pentágono regular ABCDE. ¿Cuál es la medida del ángulo AFC? A) 108º B) 112º C) 116º D) 126º E) 132º Colocamos algunos de los dígitos de 1 a 9 en las casillas en blanco de forma que no utilizamos el mismo dígito más de una vez, siendo la suma de los dígitos de cada fila y columna la que te mostramos. ¿Qué número aparece en la casilla con asterisco?

A) 1 B) 3 C) 5 D) 7 E) 9 Si el radio de la circunferencia de la figura es 4 cm, ¿cuál es, en cm, la longitud de la cuerda PR? A) 24 B) 2 C) 22 D) 22 + E) 24 −

El dibujo que ves muestra un semicírculo y un triángulo isósceles, ambos de igual área. ¿Cuál es el valor de tg x º ?

A) 1 B) 23 C)

3π D)

π2 E)

Recuerda que 1)2()1(! L−−= nnnn . Si dividimos 2007! entre 2008, el resto r de la división verifica que: A) 0=r B) 2006100 ≤≤ r C) 2007=r D) 1001 <≤ r E) Nada de lo anterior

En el triángulo rectángulo ABC (de ángulo recto en A), las bisectrices de los ángulos agudos B y C se cortan en el punto P. Si P dista 8 unidades de la hipotenusa, ¿cuántas unidades dista P de A?

A) 8 B) 3 C) 10 D) 12 E) 4

8

9

10

11

12

13

14

c

d

c

d

c

d

c

d

A

B

CD

E

F

12 7 * 13

4 16 12

R

P 45º

7

c

d

Page 19: COMPENDIUM OMEM - Toomates

¿Cuántos enteros n con 5001 ≤≤ n son divisibles por 2 pero no por 3?

A) 83 B) 84 C) 166 D) 167 E) 417 En el dibujo de la derecha, los puntos B, C y D son vértices de un octógono regular inscrito en una circunferencia de centro O y radio 1. ¿Cuál es el área del cuadrilátero BCDO?

A) 1 B) 22 C)

422 + D)

824 + E)

221+

En la figura que ves, no hecha a escala, los ángulos A1, B1, B2, C1 y C2 verifican: A1<60º B1=2B2 C1 = 2C2. ¿Cuál es el valor del ángulo x?

A) 123º90 A− B) 1º90 A− C) 13º180 A−

D) 1A E) 12º180 A− Representamos por [a] el mayor entero menor o igual que a. Así pues [2008]=2008, [π]=3, [–5,76]=–6. ¿Cuántas soluciones reales tiene la ecuación |2|]2[ 22 xx −=− ?

A) 0 B) 2 C) 3 D) 5 E) Infinitas

Dividimos el diámetro AC de una circunferencia en 4 partes iguales con los puntos P, M y Q y dibujamos una cuerda que, pasando por P, corte a

la circunferencia en B y D, siendo APPD23

= . Si el área del triángulo

ABP es 1, ¿cuál es el área del cuadrilátero ABCD?

A) 7 B) 7,75 C) 8 D) 8,5 E) 9,25

Metemos en una bolsa todos los dígitos utilizados al numerar las 320 páginas de un libro. ¿Cuál es la probabilidad de que al extraer un dígito al azar de esa bolsa resulte ser el 1?

A) 101 B)

10011 C)

21343 D)

24043 E)

21340

De los seis polinomios siguientes, ¿cuántos dividen al polinomio xx −7 ?

12 ++ xx 13 −x 12 −x 124 ++ xx xx +4 xx −2

A) 2 B) 3 C) 4 D) 5 E) 6 En cada una de estas doce casillas hay un número positivo. En la cuarta hay un 4 y en la última un 12. Si la suma de cualesquiera tres casillas consecutivas es 333, ¿qué número hay colocado en la séptima casilla?

A) 4 B) 7 C) 12 D) 317 E) Faltan datos para determinarlo

Si 1)( −= xxf y 1))(( 2 −= xxfg o , )3(g es igual a:

A) 3 B) 4 C) 8 D) 9 E) 15

4 ? 12

16

17

18

19

20

21

22

23

15

B

C D

O

A1

C1

B1

C2

x

B2

C

B

MA

QP

D

Page 20: COMPENDIUM OMEM - Toomates

La longitud de la diagonal más corta de un polígono regular de n lados inscrita en una circunferencia de radio 1 es:

A) n

sen π B) n

senπ2 C) n

sen π2 D) n

sen π22 E) n

tg π2

En un juicio se quiere determinar la culpabilidad o no de cuatro personas: P, Q, R y S. Se sabe que: 1) Si P es culpable, lo es Q. 2) Si Q es culpable, es imposible que se verifique simultáneamente que R es inocente y P culpable. 3) Si S es inocente, P es culpable y R inocente. 4) Si S es culpable, lo es P. ¿Cuántos resultaron ser culpables?

A) 1 B) 2 C) 3 D) 4 E) No se puede determinar con estos datos En un triángulo de lados 3, 4 y 5, ¿cuál es la distancia entre los pies de la altura y la mediana trazada desde el vértice opuesto al lado más largo?

A) 21 B)

107 C)

22 D)

43 E) 1

¿Cuántas soluciones en el intervalo ⎥⎦⎤

⎢⎣⎡

2,0 π tiene la ecuación

21cos =+ xxsen ?

A) 0 B) 1 C) 2 D) 3 E) Nada de lo anterior

Las dimensiones del trapecio rectángulo de la figura son las que se indican en ella. Las áreas de los cuatro triángulos que determinan las diagonales están en la relación:

A) 1, 1, 2, 4 B) 1, 1, 3, 4 C) 1, 2, 3, 4 D) 1, 2, 2, 4 E) 2, 2, 3, 4 Sean x e y dos números reales cuya suma es 1 y sean yxA += 2 , 2yxB += . Considera las afirmaciones siguientes: I. BA = para cualesquiera x e y. II. BA ≠ para algunos x e y. III. 1≤A para cualesquiera x e y. IV. 1>A para algunos x e y. Las afirmaciones verdaderas son: A) I B) II C) I y III D) I y IV

E) II y III Un semicírculo está inscrito en un triángulo equilátero como se muestra en la figura. ¿Qué fracción del triángulo cae dentro del semicírculo?

A) 4π B)

52π C)

2π D)

83π E)

52π

24

25

26

27

28

29

30

4

5

8

3

B

O

Page 21: COMPENDIUM OMEM - Toomates
Page 22: COMPENDIUM OMEM - Toomates

REAL SOCIEDAD MATEMÁTICA ESPAÑOLA

XLVI OLIMPIADA MATEMÁTICA ESPAÑOLA Comunidad de Madrid

Primera sesión, viernes 27 de noviembre de 2009

• En la hoja de respuestas, escribe la letra que corresponde a la opción que creas correcta

en cada pregunta. Si decides cambiarla, táchala con una cruz y escribe otra. • Cada respuesta correcta te aportará 5 puntos, cada respuesta en blanco 2 puntos y cada

respuesta errónea, 0 puntos. • No están permitidas calculadoras ni ningún instrumento de medida. • Tiempo: 3 horas.

En una lista de siete números, cualesquiera cuatro adyacentes suman 16 y cualesquiera cinco adyacentes suman 19. ¿Cuál es la suma de esos siete números? A) 21 B) 25 C) 28 D) 32 E) 35 En el dibujo de la derecha, que no está hecho a escala, se verifica que TSQTPT == , SRQS = y el ángulo

º20ˆ =TQP . ¿Cuál es el valor de x? A) 20 B) 25 C) 30 D) 35 E) 40 En este cuadrado mágico, el producto de los números de cada fila, columna y diagonal es el mismo. ¿Cuál es valor de sr + ?

A) 21 B)

169 C)

45 D)

1633 E) 24

En un reloj digital, como el del dibujo, en el que aparecen las horas, minutos y segundos, ¿cuántas veces cambian los seis dígitos simultáneamente en 24 horas?

A) 0 B) 1 C) 2 D) 3 E) 4

El diagrama muestra un cuadrado de lado y que se ha dividido en un cuadrado de lado x y cuatro rectángulos iguales. ¿Cuál es la longitud del lado largo del rectángulo?

A) 2

xy − B) 32xy + C) xy − D)

32y E)

2xy +

1

2

3

4

5

p q r

s 1 t

u 4 1/8

16 : 34 : 56

x

y

P

Q

R T S

20º xº

Page 23: COMPENDIUM OMEM - Toomates

En el diagrama que ves, podemos leer dos números de tres cifras cada uno: leyendo de izquierda a derecha y leyendo de arriba a abajo. Hay un único valor del dígito d para el que ambos números son primos. ¿Cuál es este valor? A) 4 B) 5 C) 6 D) 7 E) 8

Una lista de diez números está formada por 0, 1, 2, 3, 4, cada uno de ellos dos veces. Los ceros están juntos; los unos separados por 1 número; los doses separados por 2 números; los treses separados por 3 números; y los cuatros separados por 4 números. Si la lista empieza por 3, 4, …, ¿qué número aparece en último lugar?

A) 0 B) 1 C) 2 D) 3 E) 4 Las páginas de un libro están numeradas 1, 2, 3, … Si se han empleado 852 dígitos para numerarlas, ¿cuál es el número de la última página?

A) 215 B) 314 C) 320 D) 329 E) 422

¿Para cuántos valores de n se verifica que 2n y 2n son números de tres cifras?

A) 0 B) 150 C) 200 D) 300 E) 500

Si la suma de tres números primos diferentes es 40, ¿cuál es la diferencia entre los dos mayores?

A) 8 B) 12 C) 16 D) 20 E) 24 En un festival de Navidad, la entrada infantil cuesta 4,20 € y la de adulto 7,70 €. Un grupo de niños y adultos fue al festival y pagaron entre todos c €. De los siguientes números, ¿cuál de ellos es un posible valor para c?

A) 91 B) 92 C) 93 D) 94 E) 95

En el triángulo ABC de la figura, se verifica que ºˆ α=CAB y

ºˆ β=CBA , donde βα < . Si CD es la bisectriz del ángulo BCA ˆ y CE es perpendicular a AB, ¿cuál es el valor del

ángulo ECD ˆ ?

A) 2

)(180 βα +− B) 2

αβ − C) 22βα + D)

22360 βα −− E)

2βα +

Con centro en los vértices y puntos medios de los lados de un cuadrado de perímetro 8, hemos construido la región sombreada que observas, en la que todas las circunferencias son iguales. ¿Cuál es el perímetro de dicha región?

A) π B) π2 C) 8 D) π3 E) π4

¿Cuántos conjuntos de tres primos distintos tienen la propiedad de que el producto de los tres es cinco veces su suma?

A) 0 B) 1 C) 2 D) 4 E) 7

6

8

9

10

11

12

13

14

7

5 1 d 3 7

A E B

C

D α º β º

Page 24: COMPENDIUM OMEM - Toomates

A B

D

E

P

F es el conjunto de todos los números de cinco cifras en los que el producto de éstas es 15. T es el conjunto de todos los números de cinco cifras en los que el producto de éstas es 25. ¿Cuál de las siguientes afirmaciones es verdadera? A) El conjunto F tiene el doble de elementos que el conjunto T. B) El conjunto F tiene la mitad de elementos que el conjunto T. C) El conjunto F tiene 5/3 de los elementos del conjunto T. D) El conjunto F tiene 3/5 de los elementos del conjunto T. E) Ambos conjuntos tienen el mismo número de elementos. En una clase el número de chicas es más del 45% pero menos del 50% del total. ¿Cuál es el menor número posible de chicas en esa clase?

A) 3 B) 4 C) 5 D) 6 E) 7

La figura muestra un polígono regular de nueve lados en el que hemos prolongado los lados AB y DE hasta que se junten en el punto P. ¿Cuál es el valor del ángulo DPB ˆ ?

A) 40º B) 45º C) 50º D) 55º E) 60º El dibujo muestra cuatro circunferencias, iguales dos a dos, con centros en los vértices de un cuadrado y tangentes entre sí. Si el radio de las circunferencias pequeñas es 1 cm, ¿cuál es, en cm, el radio de las grandes?

A) 21+ B) 5 C) 2 D) 25 E)

34π

¿Cuántos números hay, de 10 cifras cada uno, formados solamente por unos, doses y treses y tal que cualquier par de cifras adyacentes difieran en 1?

A) 16 B) 32 C) 64 D) 80 E) 100

¿Cuál es el máximo número de cifras que puede tener un número si cualquier número formado por dos cifras consecutivas es un cuadrado perfecto?

A) 3 B) 4 C) 5 D) 6 E) 10

En un triángulo, dos de sus medianas, de longitudes 8 y 12 cm, son perpendiculares. ¿Cuál es, en cm2, el área de dicho triángulo?

A) 24 B) 32 C) 48 D) 64 E) 96 En la figura observas una circunferencia inscrita en un triángulo y circunscrita en otro. ¿Cuál es el valor de x?

A) 55 B) 60 C) 65 D) 70 E) 75

16

17

18

19

20

21

22

15

55º

Page 25: COMPENDIUM OMEM - Toomates

En una circunferencia de centro O marco los puntos A y B siendo el ángulo º60ˆ =BOA . Una segunda circunferencia es tangente interior a aquella y además tangente a los segmentos OA y OB. ¿Cuál es el cociente entre las áreas de los círculos pequeño y grande?

A) 161 B)

91 C)

81 D)

61 E)

41

¿Cuál es el área de la región formada por los puntos (x, y) tales que 372183 ≤++− yx ?

A) 3 B) 27 C) 4 D)

29 E) 5

¿Cuál es el coeficiente de x28 en el producto de polinomios

( ) ( )2142272 ...1...1 xxxxxx ++++⋅++++ ?

A) 195 B) 196 C) 224 D) 378 E) 405 Colocamos alineadas y al azar tres bolas rojas, dos blancas y una azul. ¿Cuál es la probabilidad de que no haya dos del mismo color juntas?

A) 121 B)

101 C)

61 D)

31 E)

21

En el interior de un rectángulo de dimensiones a y b con a > b, coloreamos otro rectángulo de lados paralelos a los de aquel y que deja un pasillo sin colorear de anchura uniforme de 1 cm. Si el rectángulo coloreado tiene la mitad de área del rectángulo original, ¿cuántas posibilidades hay para el par ordenado (a, b)?

A) 1 B) 2 C) 3 D) 4 E) 5

En el cuadrilátero ABCD con AB = BC = CD, los ángulos en A y en C miden 70º y 170º respectivamente. ¿Cuál es la medida, en grados, del ángulo en A?

A) 75 B) 80 C) 85 D) 90 E) 95 Una encuesta demuestra que el 70% de los encuestados aprueban una determinada medida del gobierno. En tres ocasiones, elegimos un encuestado al azar. ¿Cuál es la probabilidad de que solo en una de estas tres ocasiones el encuestado sea de los que aprueban la medida del gobierno?

A) 0,063 B) 0,0189 C) 0,233 D) 0,333 E) 0,441 El cuadrado ABCD, de área 36, verifica que el lado AB es paralelo al eje horizontal. Si los vértices A, B y C están respectivamente en las gráficas de xy alog= , xy alog2= e xy alog3= , ¿cuál es el valor de a?

A) 6 3 B) 3 C) 3 6 D) 6 E) 6

23

24

25

26

27

28

29

30

Page 26: COMPENDIUM OMEM - Toomates

XLVI OLIMPIADA MATEMÁTICA ESPAÑOLA Comunidad de Madrid

Primera sesión, viernes 27 de noviembre de 2009

TABLA DE RESPUESTAS

Pregunta Respuesta Pregunta Respuesta 1 B 16 C 2 D 17 E 3 B 18 A 4 D 19 C 5 E 20 C 6 D 21 D 7 B 22 D 8 C 23 B 9 B 24 A

10 E 25 C 11 A 26 C 12 B 27 B 13 D 28 ANULADA 14 B 29 ANULADA 15 A 30 A

Page 27: COMPENDIUM OMEM - Toomates

REAL SOCIEDAD MATEMÁTICA ESPAÑOLA

XLVII OLIMPIADA MATEMÁTICA ESPAÑOLA

Comunidad de Madrid

Primera sesión, viernes 26 de noviembre de 2010

En la hoja de respuestas, escribe la letra que corresponde a la opción que creas correcta

en cada pregunta. Si decides cambiarla, táchala con una cruz y escribe otra.

Cada respuesta correcta te aportará 5 puntos, cada respuesta en blanco 2 puntos y cada

respuesta errónea, 0 puntos.

No están permitidas calculadoras ni ningún instrumento de medida.

Tiempo: 3 horas.

Un tetrabrick contiene leche con un 2% de grasa, que supone un 40% menos de grasa que la

contenida en otro de leche entera. ¿Cuál es el porcentaje de grasa de la leche entera?

A) 5

12% B) 3% C)

3

10% D) 3,8% E) 4,2%

El triángulo ABC es rectángulo en B, y el punto D es el pie de la altura

que parte de B. Si 3AD y 4DC , ¿cuál es el área de dicho triángulo?

A) 34 B) 37 C) 21 D) 314 E) 42

Sean a, b, c, d, números reales tales que 2ba , 3 cb y 4 dc . ¿Cuál es la suma

de todos los posibles valores de da ?

A) 9 B) 12 C) 15 D) 18 E) 24

Durante un determinado año, el precio de la gasolina subió un 20% en enero; bajó un 20% en

febrero; subió un 25% en marzo; y bajó un x% en abril. Si al final de abril, el precio de la

gasolina era igual que a comienzos de enero, el entero más próximo a x es:

A) 12 B) 17 C) 20 D) 25 E) 35

Los segmentos BD y AE se cortan en C, cumpliéndose que

CECDBCAB y BA ˆ2

5ˆ . ¿Cuál es la medida del ángulo D?

A) 52,5º B) 55º C) 57,5º D) 60º E) 62,5º

1

2

3

4

5

A

B C

D 3

4

A

B

C

D

E

Page 28: COMPENDIUM OMEM - Toomates

¿Cuántos capicúas de siete cifras pueden formarse con los dígitos 2, 2, 3, 3, 5, 5, 5?

A) 6 B) 12 C) 24 D) 36 E) 48

Marcamos cuatro puntos A, B, C, D, en una recta cumpliéndose que 1 CDBCAB . En

otra paralela marcamos los puntos E y F con 1EF . Con tres de estos puntos, dos en una

recta y el tercero en la otra, formamos un triángulo. ¿Cuántos valores posibles puede tener el

área de dicho triángulo?

A) 3 B)4 C) 5 D) 6 E) 7

¿Cuál es el resto de la división de 2010210 3...333 entre 8?

A) 0 B) 1 C) 2 D) 4 E) 5

Los tres primeros términos de una progresión aritmética son 22 x , 45 x y 8x .

Si el n-ésimo término de esta progresión es 2010, ¿quién es n?

A) 256 B) 503 C) 1004 D) 1506 E) 8037

En un pentágono regular de lado 2 cm, consideramos el círculo circunscrito y el inscrito. Sea

P la diferencia de sus áreas. En un heptágono regular, también de lado 2 cm, hacemos lo

mismo y llamamos H a la diferencia de sus áreas. De las siguientes afirmaciones, ¿cuál es la

verdadera?

A) HP49

25 B) HP

7

5 C) HP D) HP

5

7 E) HP

25

49

Del triángulo ABC conocemos las coordenadas de dos vértices, )0,3(A y )3,0(B , y sabemos

que el tercer vértice, C, está en la recta 7 yx . ¿Cuál es el área de dicho triángulo?

A) 6 B) 8 C) 10 D) 12 E) 14

El quinto y el octavo términos de una progresión geométrica son los números 7! y 8!

respectivamente. ¿Quién es el primer término de esa progresión?

A) 60 B) 75 C) 120 D) 225 E) 315

En el triángulo ABC, AB = 13, AC = 15 y la longitud de la altura sobre BC es 12. ¿Cuál es la

suma de los dos valores posibles para BC?

A) 15 B) 16 C) 17 D) 18 E) 19

Si 0 < r < 3 y consideramos las cinco ecuaciones siguientes, de incógnita x, ¿en cuál de ellas

es mayor la solución?

A) 713 x

r B) 710

13

xr

C) 7213 x

r

D) 713 x

r E) 71

13

x

r

6

8

9

10

11

12

13

14

7

Page 29: COMPENDIUM OMEM - Toomates

Se juntan tres cuadrados como se indica en la figura. ¿Cuánto mide

el ángulo x?

A) 60º B) 50º C) 45º D) 40º E) 30º

Si n 20002010log10 , ¿cuál de los siguientes números es el valor de

20002010log10 ?

A) 1n B) n1 C) n

1 D) 1n E) Es imposible determinarlo con esos datos

Sean a y b los catetos de un triángulo rectángulo. Si d es el diámetro de la circunferencia

inscrita y D el diámetro de la circunferencia circunscrita, d + D es igual a:

A) a + b B) ab C) 22 ba D) ab E) 2ba

La figura muestra un rectángulo ABEF y un triángulo ABC. Se

sabe que EBCFCA ˆˆ . Si FC = 6 y CE = 2, el área del

triángulo ABC es:

A) 12 B) 16 C) 28

D) 38 E) Faltan datos para poder contestar

Un tren consta de cinco vagones: I, II, III, IV y V y una locomotora que los arrastra. ¿De

cuántas maneras se pueden distribuir los vagones de modo que el vagón I esté más cerca de la

locomotora que el vagón II?

A) 120 B) 60 C) 48 D) 30 E) 10

Dos cuadrados de lado 1 tienen un vértice común y el lado de uno de

ellos está sobre la diagonal del otro, como se muestra en la figura. ¿Cuál

es el área sombreada?

A) 12 B) 2

2 C)

2

12 D) 12 E) 23

Marcamos sobre una recta los números 5

1 y

3

1 como indica la figura, en la que hemos

señalado 16 intervalos iguales. ¿En qué posición estaría 4

1?

1/5 a b c d e 1/3

A) a B) b C) c D) d E) e

16

17

15

F C E

B A

18

19

20

21

x

Page 30: COMPENDIUM OMEM - Toomates

Q

P

R S

A

B

C 68º

D

30º 30º

36º 36º

Alicia calcula correctamente 2

6666

. ¿Cuál es la cifra de las unidades del número obtenido?

A) 1 B) 2 C) 3 D) 6 E) 8

¿Para cuántos enteros positivos n resulta que n y n3 + 3 son ambos primos?

A) 0 B) 1 C) 2 D) 3 E) Infinitos

En un viaje con bastantes atascos, Juanje alcanzó una media de 55 km/h durante las dos

primeras horas y 70 km/h el resto del viaje. Si la media total alcanzada fue de 60 km/h,

¿cuánto tiempo duró el viaje?

A) 6 h B) 4,5 h C) 4 h D) 3,5 h E) 3 h

Los triángulos PQR y PRS de la figura son isósceles con QP = QR = 9

cm y PR = PS = 6 cm. ¿Cuál es la longitud SR?

A) 1 cm B) 2 cm C) 3 cm D) 4 cm E) 5 cm

¿Cuál es la media de los dos decimales 90,2

y 908,2

?

A) 2,05 B) 2,15 C) 90,2

D) 508,2

E) 2,095

La suma de cuatro enteros consecutivos nunca puede ser:

A) 2010 B) 4006 C) 4004 D) 1002 E) 998

En el triángulo ABC, la bisectriz del ángulo B corta al lado

AC en el punto D, siendo º68ˆ CDB . ¿Cuánto vale la

diferencia entre los ángulos C y A del triángulo?

A) 44º B) 120º C) 24º

D) 30º E) No hay información suficiente

Dos botellas de igual volumen están llenas, ambas, de agua y zumo. Las razones de los

volúmenes de agua y zumo son, 2:1 y 4:1. Echamos el líquido de ambas botellas en una

garrafa. La proporción de agua y zumo en la garrafa es:

A) 3:1 B) 6:1 C) 11:4 D) 5:1 E) 8:1

¿Cuál es la medida del ángulo de la figura?

A) 110º B) 115º C) 120º

D) 126º E) 130º

29

28

26

25

24

18

19

20

21

22

24

25

26

27

28

29

30

27

30

23

22

Page 31: COMPENDIUM OMEM - Toomates

XLVII OLIMPIADA MATEMÁTICA ESPAÑOLA

Comunidad de Madrid

Primera sesión, viernes 26 de noviembre de 2010

TABLA DE RESPUESTAS

Pregunta Respuesta Pregunta Respuesta

1 C 16 B

2 B 17 A

3 D 18 D

4 B 19 B

5 A 20 A

6 A 21 A

7 A 22 E

8 E 23 B

9 B 24 E

10 C 25 D

11 A 26 E

12 E 27 C

13 D 28 A

14 B 29 C

15 C 30 D

Page 32: COMPENDIUM OMEM - Toomates

REAL SOCIEDAD MATEMÁTICA ESPAÑOLA

XLVIII OLIMPIADA MATEMÁTICA ESPAÑOLA

Comunidad de Madrid

FASE CERO: viernes 25 de noviembre de 2011 .

En la hoja de respuestas, escribe la letra de la opción que creas correcta.

Cada respuesta correcta te aportará 5 puntos; cada respuesta en blanco 1 punto, y cada

respuesta errónea, 0 puntos.

No está permitido el uso de calculadoras, instrumentos de medida o de cualquier aparato

electrónico.

TIEMPO: 3 HORAS

1. En la figura adjunta, CE y DE son cuerdas de igual longitud de la

circunferencia de centro O de la que CD es un diámetro. Si el arco AB es un

cuarto de circunferencia, ¿cuál es el cociente entre las áreas de los triángulos

CED y AOB?

A) 2 B) 3 C) 4 D) 3 E) 2

2. En una carrera de 10km, Víctor supera a Segundo en 2km y a Lentini en 4km. Si los

corredores mantienen una velocidad constante a lo largo de toda la prueba, ¿cuántos

kilómetros le sacará Segundo a Lentini?

A) 2 B) 4

9 C)

2

5 D)

4

11 E) 3

3. A partir de un triángulo rectángulo ABC de hipotenusa AB, construimos otro triángulo

rectángulo ABD con la misma hipotenusa AB. Si BC = 1, AC = b y AD = 2, BD es igual a:

A) 12 b B) 32 b C) 12 b +2 D) b2 + 5 E) 32 b

4. Pedro anduvo una determinada distancia a velocidad constante. Si hubiera ido 0,5 km/h más

rápido, habría recorrido la misma distancia en 5

4 del tiempo original, pero si hubiera ido

0,5km/h más despacio, habría tardado 2

5 de hora más. ¿Cuál fue – en kilómetros – la

distancia recorrida por Pedro?

A) 2

27 B) 15 C)

2

35 D) 20 E)

25

5. En el triángulo ABC de la figura, el ángulo C es de 90o, D es el punto

medio de AB, DE es perpendicular a AB y AB = 20, AC = 12. ¿Cuál es el

área del cuadrilátero ADEC?

A) 75 B) 58,5 C) 48 D) 37,5

E) Nada de lo anterior

Page 33: COMPENDIUM OMEM - Toomates

6. Hay dos números positivos que, colocados entre el 3 y el 9, hacen que los tres primeros estén

en progresión geométrica y los tres últimos en progresión aritmética. La suma de estos dos

números positivos es:

A) 2

27 B)

4

45 C)

2

21 D) 10 E)

2

19

7. Consideramos dos circunferencias: la mayor, de centro P y radio R; la otra, de centro Q y

radio r, y dibujamos el segmento PQ. ¿Cuál de las siguientes afirmaciones es falsa?

A) R – r puede ser igual a PQ. B) R + r puede ser igual a PQ

C) R + r puede ser menor que PQ D) R – r puede ser menor que PQ

E) Nada de lo anterior.

8. Al colorear n bolas, numeradas de 1 a n, en rojo y en negro, resulta que entre las primeras 50

hay 49 rojas y entre las restantes 7 de cada 8 son rojas. Si el número de bolas rojas es mayor

o igual que el 90% del total de bolas, el valor máximo de n es:

A) 225 B) 210 C) 200 D) 180 E) 175

9. En el triángulo PQR de la figura, RS es bisectriz del ángulo R y D está

en la prolongación de PQ de modo que el ángulo n es recto. Entonces:

A) QPm 2

1 B) QPm

2

1 C) QPD

2

1

D) mD2

1 E) Nada de lo anterior es correcto.

10. El resto de la división del polinomio p (x) = x100

entre x2 - 3x + 2 es:

A) 2200

– 1 B) 2100

(x – 1) – (x – 2) C) 2100

(x – 3) D) (2100

– 1)x + 2(299

– 1)

E) 2100

(x + 1) – (x + 2)

11. En el rombo ABCD dibujamos segmentos paralelos a la diagonal BD y de extremos en los

lados del rombo. Consideremos una gráfica que muestre la posible longitud de cada

segmento en función de su distancia al vértice A. La gráfica es:

A) Una recta que pasa por el origen B) Una recta que corta a los semiejes positivos

C) Dos segmentos formando una V D) Dos segmentos formando una V invertida

E) Nada de lo anterior

12. ¿Cuál de los siguientes números es la suma de 11 enteros consecutivos?

A) 7 B) 77 C) 777 D) 7770 E) 7771

13. Si el cociente entre las medidas de los dos catetos de un triángulo es 2

1, el cociente entre las

medidas de los correspondientes segmentos de hipotenusa determinados por la altura sobre la

misma es:

A) 4

1 B)

23

1

C)

152

1

D)

52

1

E)

5

1

Page 34: COMPENDIUM OMEM - Toomates

14. Si S es la suma de los restos de la división de 30, 31, 32, 33, 34 y 35 entre 6, ¿cuál es el resto

de la división de S entre 6?

A) 0 B) 1 C) 2 D) 3 E) 5

15. Isa sale de excursión cuando las agujas de su reloj están juntas entre las 8 y las 9 y llega a su

destino entre las 2 y las 3, cuando las agujas forman un ángulo de 180o. ¿Cuánto duró su

excursión?

A) 6 horas B) 6 horas y 43 +11

7 minutos C) 5 horas y 16+

11

4minutos

D) 6 horas y media E) Nada de lo anterior

16. Si x e y son números reales con 0 ,3 3 xyxyx , ¿cuál es el entero más próximo a

x – y?

A) -3 B) -1 C) 2 D) 3 E) 5

17. En la circunferencia de la figura, de centro O, prolongamos la cuerda

AB hasta que corte en C al diámetro OD, de manera que BC sea

igual al radio, y llamamos y a los ángulos que se indican. ¿Cuál

de las siguientes afirmaciones expresa la relación entre y ?

A) = 3 B) = 2 C) = 60o

D) No hay relación especial entre y

E) = 3 o = 2 según la longitud de AB

18. Si 1 < x < 10, 1< y < 10, ¿qué afirmación de las siguientes es necesariamente verdadera?

A) x

y10

B) 10

yx C)

10

xy D)

10

yx E) y

x

100

19. El número de triángulos con los tres lados desiguales, de longitud entera y de perímetro

menor que 13 es:

A) 1 B) 2 C) 3 D) 4 E) 18

20. La función 32

187)(

n

nnf toma valores enteros para ciertos valores enteros de n. La suma

de todos estos )(nf enteros es:

A) 14 B) 21 C) 24 D) 28 E) 30

21. Si P es un punto interior al rectángulo ABCD tal que PA = 3 cm, PD = 4 cm,

PC = 5 cm, PB, en centímetros, es igual a:

A) 32 B) 23 C) 33 D) 24 E) 2

22. Si a, b, c, d y e son enteros distintos y 1244444 edcba ,

edcba es igual a:

A) 12 B) 16 C) 17 D) 24 E)32

Page 35: COMPENDIUM OMEM - Toomates

23. Si la base mayor de un trapecio isósceles es igual a la diagonal y la base más pequeña igual a

la altura, el cociente entre la base pequeña y la grande es:

A) 2

1 B)

3

2 C)

4

3 D)

5

3 E)

5

2

24. Pedro tiene una caja fuerte con un código de tres cifras. Ha olvidado el código, pero sabe que

las tres cifras son diferentes y que la primera cifra es igual al cuadrado del cociente de la

segunda entre la tercera cifra. ¿Cuántas combinaciones deberá probar, en el peor de los

casos, hasta encontrar el código?

A) 1 B) 2 C) 3 D) 4 E) 8

25. Dos segmentos verticales de 20cm y 80cm (están apoyados sobre el suelo) y separados 1 m.

El punto de intersección de las rectas que unen el punto de más altura de un segmento con el

más bajo del otro (el extremo superior de un segmento con el inferior del otro) está a una

altura de

A) 50cm B) 40cm C) 16cm D) 60cm E) Nada de lo anterior

26. En una caja hay 9 tarjetas numeradas del 1 al 9. Antonio y Beatriz sacan cada uno al mismo

tiempo una tarjeta de la caja. ¿Cuál es la probabilidad de que el número de la tarjeta de

Antonio sea el doble o más que el número de la tarjeta de Beatriz?

A) 18

7 B)

9

4 C)

81

28 D)

18

5 E)

3

1

27. El segmento que une los puntos medios de las diagonales de un trapecio tiene 3 unidades de

longitud. Si la base mayor mide 97 unidades, la base menor mide:

A) 94 B) 92 C) 91 D) 90 E) 89

28. ¿Cuántos divisores de cuatro cifras tiene el número 1022?

A) 2 B) 3 C) 4 D) 5

E) 6

29. En el dibujo que ves, AB y AC son lados del triángulo isósceles ABC

en el que inscribimos el triángulo equilátero DEF. Si los ángulos a, b y

c son los que hemos marcado, entonces:

A) cab 2

1 B) cab

2

1 C) cba

2

1

D) cba 2

1 E) Nada de lo anterior

30. Escribimos en la pizarra todos los enteros del 1 al 2011. María subraya los múltiplos de 2,

luego los múltiplos de 3, y luego los múltiplos de 4. ¿Cuántos números ha subrayado

exactamente dos veces?

A) 1005 B) 1004 C) 503 D) 336 E)169

Page 36: COMPENDIUM OMEM - Toomates

XLVIII Olimpiada Matematica Espanola

Primera Fase

Primera sesion

Viernes tarde, 16 de diciembre de 2011

OlimpiadaMatemáticaEspañola RSME

1. Sean a, b y c tres numeros reales positivos cuyo producto es 1. Demostrarque si la suma de estos numeros es mayor que la suma de sus recıprocos,entonces exactamente uno de ellos es mayor que 1.

2. En un triangulo rectangulo de hipotenusa unidad y angulos respectivos de30◦, 60◦ y 90◦, se eligen 25 puntos cualesquiera. Demostrar que siemprehabra 9 entre ellos que podran cubrirse con un semicırculo de radio 3/10.

3. Sea P un punto interior a un triangulo ABC y sean HA, HB , HC los orto-centros de los triangulos PBC, PAC y PAB, respectivamente. Demostrarque los triangulos HAHBHC y ABC tiene igual area.

No esta permitido el uso de calculadoras.Cada problema se puntua sobre 7 puntos.

El tiempo de cada sesion es de 3 horas y media.

Page 37: COMPENDIUM OMEM - Toomates

XLVIII Olimpiada Matematica Espanola

Primera Fase

Segunda sesion

Sabado manana, 17 de diciembre de 2011

OlimpiadaMatemáticaEspañola RSME

4. Sea ABCD un cuadrilatero convexo y P un punto interior. Determinar quecondiciones deben cumplir el cuadrilatero y el punto P para que los cuatrotriangulos PAB, PBC, PCD y PDA tengan la misma area.

5. Sean a, b y c las longitudes de los lados de un triangulo ABC. Si

b(a + b)(b + c) = a3 + b(a2 + c2) + c3,

demostrar que la medida (en radianes) de los angulos A, B y C cumple larelacion

1√A +

√B

+1√

B +√

C=

2√A +

√C

.

6. Tenemos una coleccion de esferas iguales que apilamos formando un tetraedrocuyas aristas tienen todas n esferas. Calcular, en funcion de n, el numerototal de puntos de tangencia (contactos) entra las esferas del monton.

No esta permitido el uso de calculadoras.Cada problema se puntua sobre 7 puntos.

El tiempo de cada sesion es de 3 horas y media.

Page 38: COMPENDIUM OMEM - Toomates

XLVIII OLIMPIADA MATEMÁTICA ESPAÑOLA

Primera fase Segunda sesión

Sábado 17 de diciembre de 2011

4. Dado un entero positivo n, hallar la suma de todos los enteros positivos

inferiores a 10n que no son múltiplos de 2 ni de 5.

5. Sea ABCD un cuadrilátero convexo y P un punto interior. Determinar qué

condiciones deben cumplir el cuadrilátero y el punto P para que las áreas de los

triángulos PAB, PBC, PCD y PDA sean iguales.

6. Sean a, b y c las longitudes de los lados de un triángulo ABC. Si

3223 ccabacbbab ,

demostrar que la medida (en radianes) de los ángulos A, B y C cumple la

relación

CACBBA

211.

No está permitido el uso de calculadoras

Cada problema se puntúa sobre 7 puntos

El tiempo de cada sesión es de 3 horas

Page 39: COMPENDIUM OMEM - Toomates

D C

B A

REAL SOCIEDAD MATEMÁTICA ESPAÑOLA

XLIX OLIMPIADA MATEMÁTICA ESPAÑOLA

Comunidad de Madrid

FASE CERO: viernes 23 de noviembre de 2012

En la hoja de respuestas, escribe la letra de la opción que creas correcta

Cada respuesta correcta te aportará 5 puntos; cada respuesta en blanco 1 punto, y cada respuesta errónea, 0

puntos.

No está permitido el uso de calculadoras, instrumentos de medida o de cualquier aparato electrónico.

TIEMPO: 3 horas.

1. ¿Cuántos números reales satisfacen la ecuación 2222 1524 xxx

A) 0 B) 1 C) 2 D) 3 E) 4

2. ¿Cuál es el menor de los siguientes números?

A) 2

3 B) 2log3 C)

2

D) 10log4 E) 3 4

3. El cuadrilátero ABCD es circunscrito a una circunferencia. Si AB = 16 y CD = 10, ¿cuál es el perímetro del

cuadrilátero? A) 50 B) 52 C) 54 D) 56 E) 58

4. Si 9x y 12y , ¿cuál es el mayor valor posible para yx ?

Recuerda: [a] es el mayor entero menor o igual que a. A) 268 B) 225 C) 242 D) 270 E) 256

5. Si a y b son enteros positivos para los que 201722 ba , ¿cuál es el valor

de 22 ba ?

A) 2 026 081 B) 2 026 082 C) 2 026 083 D) 2 029 545 E) 2 034 145

6. Dos rectas perpendiculares, que se cortan en el centro de un círculo de radio 1, dividen a éste en cuatro partes iguales. En una de estas partes inscribimos una circunferencia, como se muestra en la figura. ¿Cuál es su radio?

A) 3

1 B)

5

2 C) 12 D)

2

1 E) 22

7. Si x e y son números distintos de cero tales que yxy

xyx · , ¿cuál es

el valor de x + y?

A) 2

3 B)

2

1 C) 0 D)

2

1 E)

2

3

Page 40: COMPENDIUM OMEM - Toomates

8. a, b y c son números positivos que verifican ,2922 acba

252,182 22 bcacabcb . ¿Cuál es el valor de a + b + c?

A) 4 B) 5 C) 6 D) 7 E) 8

9. Una función f, definida para cualquier número distinto de cero, verifica que

xxfxx

f 2)(·11

. Calcula f(2).

A) 2,5 B) 3 C) 3,5 D) 4 E) 4,5

10. N = [xyz] es un número de tres cifras, todas distintas de cero.

Si N 2

= (x + y + z) 5

entonces 222 zyx es igual a:

A) 21 B) 23 C) 29 D) 33 E) 37

11. En el triángulo rectángulo ABC de hipotenusa AB, el cateto AC mide 15. Si la altura CH divide a AB en dos segmentos AH y HB, con HB = 16, el área del triángulo ABC es:

A) 120 B) 144 C) 150

D) 216 E) 5144

12. Una bolsa contiene 11 bolas numeradas con: 1, 2, 3,…, 11. Sacamos simultáneamente seis bolas al azar. ¿Cuál es la probabilidad de que la suma de los números de estas seis bolas sea impar?

A) 231

133 B)

231

115 C)

2

1 D)

231

118 E)

11

6

13. El número de soluciones enteras positivas del sistema

23

44

yzxz

yzxy es:

A) 0 B) 1 C) 2 D) 3 E) 4

14. En el triángulo rectángulo ABC el ángulo  = 30º. La circunferencia de

diámetro el cateto AB corta a la hipotenusa en un punto D. Si 3CD ,

¿cuánto mide el cateto AB?

A) 33 B) 6 C) 34 D) 8 E) 35

15. El discriminante de una ecuación de segundo grado con coeficientes enteros nunca puede ser:

A) 23 B) 24 C) 25 D) 28 E) 33 (Recuerda: el discriminante de la ecuación ax

2 + bx + c = 0 es b

2 – 4ac)

16. Lanzamos ocho veces un dado equilibrado de seis caras. Si el número 3 aparece exactamente 3 veces, ¿cuál es la probabilidad de que no aparezca dos veces consecutivas?

A) 14

5 B)

7

3 C)

2

1 D)

7

4 E)

14

9

16

H

C

B A

15

Page 41: COMPENDIUM OMEM - Toomates

17. Si a, b, c son distintos de cero las soluciones de la ecuación ax 2 + bx + c = 0

vienen dadas por la expresión:

A)

acbb

c

4

2

2

B)

acbb

c

4

2

2

C) a

acbb

2

42

D)a

acbb

2

42 E) Nada de lo anterior

18. La figura adjunta muestra el triángulo equilátero ABC, su circunferencia inscrita y el segmento DE perpendicular al lado AC y tangente a la circunferencia inscrita; el punto D sobre el lado AB y el E sobre el lado AC. Si AE = 1, la longitud del lado del triángulo ABC es:

A) 33 B) 33 C) 36 D) 2

32 E) 36

19. La circunferencia de la figura está inscrita en el cuadrilátero ABCD, siendo R, S, T y U los puntos de tangencia con los lados. Si  = 90º, DR = 3 y el arco RST es de 210º, el área del círculo es:

A) 36 B) 32 C) 27 D) 18 E) Nada de lo anterior

20. Determina el número n, de manera que los últimos siete dígitos de n! sean 8 000 000.

A) 24 B) 25 C) 26 D) 27 E) 28

21. Desde el punto medio, M, del segmento AB, de p unidades de longitud, trazamos el segmento MR, de q unidades de longitud, perpendicular a AB. Si

2

pRT , las longitudes de los

segmentos AT y TB son las soluciones de la ecuación:

A) x 2

+ px + q 2 = 0 B) x

2 - px + q

2 = 0

C) x 2

+ px - q 2

= 0 D) x 2

- px - q 2

= 0 E) x 2

- px + q = 0

22. Los tres cuadrados de la figura son iguales y de lado 1. ¿Cuál es el área del triángulo ABC?

A) 6

1 B)

5

1 C)

9

2 D)

3

1

E) 4

2

23. El valor de 222 004112220201007112220201·2010112220201 es:

A) 48 B) 38 C) 28 D) 18 E) 8

E

D

C B

A

U

T

S

R

D

C

B A

R

T M B A

C

B A

Page 42: COMPENDIUM OMEM - Toomates

24. La base de un triángulo isósceles mide 2 . Si las medianas sobre los dos

lados iguales son perpendiculares entre sí, el área del triángulo es: A) 1,5 B) 2 C) 2,5 D) 3,5 E) 4

25. En el triángulo ABC, BD es una mediana y E su punto medio. Si la

prolongación de CE corta a AB en F con BF = 5, la longitud del lado AB es: A) 11 B) 12 C) 13 D) 14 E) 15

26. Sobre dos de los lados del cuadrado ABCD de la figura, se construyen dos triángulos isósceles e iguales, AEB y BCF con uno de sus ángulos de 120º.

Si 2EF , el área del cuadrado ABCD es:

A) 22 B) 3 C) 32 D) 4

E) Nada de lo anterior

27. Si los lados de un triángulo isósceles, no rectángulo, son: senx, cosx y tgx, el valor de senx es:

A) 2

15 B)

4

15 C)

2

1 D)

2

3 E) Nada de lo

anterior

28. Colocados en orden creciente los números a = 1000!, b = (400!)·(400!)·(200!), c = (500!)·(500!), d = (600!)·(300!)·(100!) y e = (700!)·(300!), la respuesta

correcta sería: A) a < b < c < d < e B) b < c < d < e < a C) b < d < c < e < a

D) b < d < c < a < e E) c < b < a < d < e

29. Elegido al azar un número x en el intervalo [0, 3], la probabilidad de que el número elegido verifique 15x

2 + 3 < 14x es:

A) 5

3 B)

15

4 C)

45

4 D)

9

1 E)

3

1

30. El valor de la suma de la siguiente serie de infinitos sumandos es:

...2048

1

1024

1

512

1

256

1

128

1

64

1

32

1

16

1

8

1

4

1

2

11 S

A) 0 B) 7

2 C)

7

6 D)

32

9 E)

32

27

D C

F E

B A

Page 43: COMPENDIUM OMEM - Toomates

REAL SOCIEDAD MATEMÁTICA ESPAÑOLA

XLIX OLIMPIADA MATEMÁTICA ESPAÑOLA

Comunidad de Madrid

FASE CERO: viernes 23 de noviembre de 2012

01524152422222222 xxxxxx

0144

034601524·1524

2

22222

xx

xxxxxxxx

En la primera ecuación el discriminante es D = b2 – 4ac = 16 + 72 > 0, dos soluciones reales.

En la segunda D = b2 – 4ac = 16 – 16 = 0, una solución real. En total tres soluciones reales.

2

3> 1, 2log3 < 1 = 3log3 ,

2

> 1, 10log4 > 14log4 , 114 33 .

El menor de todos es 2log3 .

Considerando los puntos de tangencia, E, F, G, H, como ED = DH, HC = CG, GB = BF y FA = AE, el perímetro es igual 2(AB + DC) = 2(16 + 10) = 52.

100811099 xxx . Análogamente 169144 y .

Por lo tanto x + y < 269, luego el máximo valor de [x + y] = 268.

Es importante darse cuenta que 2017 es primo.

Como a 2 – b 2 = (a + b)(a – b) = 2017 a – b = 1 y a + b = 2017, de donde a = 1009 y b = 1008. Se deduce que a 2 – b 2 = 1018081 + 1016064 = 2034145.

OC = OB = r, 2rOA , por lo tanto 2rrAB , es decir,

1221

1211

rr

.1· yy

xyx

Si y = 1 x = x – 1 que es imposible.

Si y = – 1 – x = x + 1 2

1x . Por lo tanto

2

3 yx .

1D

2B

3B

4A

5E

6C

7A

D C

B A

H

G

F

E

A C

B

O

Page 44: COMPENDIUM OMEM - Toomates

Sumando las tres expresiones dadas,

8

9

2

28811)(72)()(

252

182

2922

2

2

2

cbacbacba

bcac

abcb

acba

Como a, b, c, son números positivos, entonces )( cba = 8.

xxfxx

f 2)(·11

.

Si tomamos 12

12)2(

2

1

ffx . Si tomamos 4)2(

2

1

2

12

ffx .

Eliminando entre ambas expresiones

2

1f se obtiene

2

9)2(9)2(·2 ff .

Si 5210100 zyxzyx quiere decir que (x + y + z) es un cuadrado perfecto y que

el número N es una quinta potencia de tres cifras. La única posibilidad es 35 = 243 y verifica 2 + 4 + 3 = 9 que es un cuadrado perfecto.

29342 222222 zyx .

9022516·)16(15 22 AHAHAHAHAH .

Se deduce que CH = 12 y el área 1502

12·25S .

Posibilidades de elegir las 6 bolas: 4626,11 C .

Para que la suma sea impar tiene que haber 1, 3 ó 5 números impares y los restantes pares. Un impar y cinco pares hay 6·C5,5 = 6.

Tres impares y tres pares hay C6,3·C5,3 = 20·10 = 200. Cinco impares y un par hay C6,5·5 = 30.

La probabilidad pedida es 231

118

462

236

462

302006

.

23

123·

z

zzyx

Si z = 1, x + y = 23.

Pero como 44· yzx , sólo hay dos posibilidades: x = 21, y = 2 o x = 1, y = 22.

Si z = 23, x + y = 1, no tiene solución con números enteros positivos.

En los triángulos 30º-60º-90º de hipotenusa 2x, las longitudes de los

catetos son, x y 3x . Por lo tanto en el triángulo CDB, CD = 3 ,

CB = 32 , DB = 3. En el triángulo ADB, DB = 3 AB = 6.

8E

9E

10C

11C

12D

13C

14B

16 H

C

B A

15

60º

60º 30º

C D

B A

Page 45: COMPENDIUM OMEM - Toomates

Veamos la paridad de b. Si b es par b 2 – 4ac = 4k (múltiplo de 4).

Si b es impar b 2 = 4k + 1 y por lo tanto b 2 – 4ac = 4k + 1. Por lo tanto el discriminante nunca puede ser 23 = 4k – 1.

Casos posibles C8,3 = 56. Casos desfavorables. 3 veces consecutivas: 6 casos (123, 234, 345, 456, 567, 678) 2 veces consecutivas y la otra no: (12, 23, 34, 45, 56, 67, 78) En cinco de ellas el 3 puede aparecer en cuatro posiciones distintas y en dos de ellas (12, 78) en cinco posiciones distintas. 5·4 + 2·5 = 30. En total 36 casos desfavorables y 20 restantes favorables.

La probabilidad es 14

5

56

20 .

acbb

c

acbba

acbb

acbb

acbb

a

acbbx

4

2

4·2

4

4

2

4

22

22

2

22

Los radios OP y OT en los puntos de tangencia determinan el cuadrado TOPE.

O además de ser el incentro es el baricentro por lo que

6

33

2

1

3

1

3

1 llBPOP

.

AP = AE + EP 333636

31

2

1 lll

ll .

Sea O el centro de la circunferencia. El triángulo ORD es 30º-

60º-90º y como DR = 3 el cateto 33OR que es el radio de

la circunferencia.

27332

S .

Como termina en seis ceros debe aparecer en n! seis veces el factor 5.

Por lo tanto 25 n < 30. Quitando los seis factores 5 y los seis factores 2, el producto de los restantes factores debe terminar en 8, es decir, 3·6·7·9·2·11·12·13·14·3·16·17·18·19·4·21·22·23·24·…. = ………………..8

Basta con ir operando con las últimas cifras de estos productos. El producto de los factores escritos termina en 4. El siguiente 4·26 termina en 4 y el siguiente 4·27 termina en 8. El número buscado es 27.

La suma es AT + TB = AB = p. Corresponde a las respuestas B) D) E). Necesitamos su producto.

15A

16A

17A

18B

19C

20D

21B

E

D

C B

A

P

O

T

U 210º

210º

T

S R

D

C

B A

30º

90º 210º

30º

Page 46: COMPENDIUM OMEM - Toomates

2222

42

1

4qpq

pMT 22 4

2

1

2

1qppAT 22 4

2

1

2

1qppBT

2

2

22

2

42

1

2

1· qqppBTAT

. Por lo tanto la ecuación es x 2 - px + q 2 = 0.

Los triángulos ABD y CBA son semejantes y además AD = 2, AB = 1,

5BD por lo tanto 5

52

5

5

15

1 ACBC

BC

El área es 5

1

5

52·

5

2

1S .

Llamando a = 201220112010, b = 201220112007, c = 201220112004, observamos que

2

cab

.

Tenemos a 2 – 2·b 2 + c 2 = 182

6)(

2

1

2

2

4

2·2

22

222

222

ca

accac

accaa .

El triángulo ABG es 45º-45º-90º y la hipotenusa mide 2 , luego los

catetos miden 1 y 1. Análogamente en el triángulo AHG la hipotenusa

mide 2 y sus catetos AH y HG, 2

2.

Como G es el baricentro, CH = 3HG = 2

23.

El área del triángulo es 2

32·

2

23·

2

1S .

DH es la paralela media y por lo tanto mediana del triángulo BCD. También CE es mediana de dicho triángulo luego G es su baricentro y por lo tanto DG = 2GH. Como los triángulos GHC y FBC son semejantes con razón de semejanza 2, al igual que los triángulos DGC y AFC, se deduce fácilmente que GH = 2,5, DG = 5, AF = 10 y en conclusión el lado AB = 15.

El triángulo EFB es rectángulo e isósceles (45º-45º-90º). Si la

hipotenusa es 2 los catetos miden 1.

En el triángulo AEB por el teorema del coseno,

32

1·1·1·211º120cos···2222

EBAEEBAEAB .

22B

24A

25E

26B

23D

E

D

C

B A

G

F

H

C

B A

D

D C

F E

B A

G

C

B A H

Page 47: COMPENDIUM OMEM - Toomates

Si senx = cosx el triángulo es 45º-45º-90º y por tanto rectángulo.

Si senx = tgx cosx = 1 con lo que senx = 0 y no tiene sentido.

Por lo tanto senxxsensenxxx

senxxtgxx 22 1cos

coscoscos . Las soluciones

de esta ecuación son 2

411 senx . La única solución válida es la positiva

2

51.

Es fácil observar que a > e porque tienen 700 factores iguales y los 300 restantes son mayores los de a. Análogamente se observa que e > c. También es fácil observar que c > b porque tienen 800 factores iguales y los 200 restantes son mayores los de c. Para situar a d lo comparamos con c.

1·501

401·400·...·

99·599

499·302·

100·600

500·301

1·...·99·100·600·...·502·501

500·...·401·400·...·302·301

!100·!300·!600

!500·!500

d

c

La primera fracción 160000

150500

100·600

500·301 . En las siguientes fracciones el denominador va

disminuyendo (evidente) mientras que el numerador va aumentando (*), luego dcd

c1

Comparamos d con b.

1600·...·402·401

200·...·102·101·400·...·302·301

!100·!300·!600

!200·!400·!400

d

b, ya que todos los factores del

numerador son menores que los del denominador. Luego b < d. El orden es pues b < d < c < e < a. (*) Si a < b entonces (a + 1)·(b – 1) = ab + b – a – 1 > ab.

Sólo se daría la igualdad si b = a + 1, que no es el caso en ninguna de las fracciones.

03141514315 22 xxxx . Las raíces de la ecuación 031415 2 xx son

3

15

3

30

414

30

18019614x . La solución de la inecuación es el intervalo

5

3,

3

1 cuya

amplitud es 15

4

3

1

5

3 . La probabilidad pedida es el cociente entre las amplitudes de los

intervalos. 45

43:

15

4 .

La suma de los términos de la progresión geométrica ,...64

1,

8

1,1 de razón

8

1r es:

7

8

8

11

1...

64

1

8

11

. Análogamente 7

4

8

11

2

1

...128

1

16

1

2

1

;

7

2

8

11

4

1

...256

1

32

1

4

1

.

La suma pedida es 7

2

7

2

7

4

7

8 .

27A

28C

29C

30B

Page 48: COMPENDIUM OMEM - Toomates

XLIX OLIMPIADA MATEMÁTICA ESPAÑOLA

FASE CERO-COMUNIDAD DE MADRID

Primera sesión, viernes 23 de noviembre de 2012

Hoja de respuestas

Nombre y apellidos: …………………………………………….…………………………. Tfno. ………………….

Centro ………………………………………………………………… Curso …….. Fecha de nacimiento ………….

1.- 16.-

2.- 17.-

3.- 18.-

4.- 19.-

5.- 20.-

6.- 21.-

7.- 22.-

8.- 23.-

9.- 24.-

10.- 25.-

11.- 26.-

12.- 27.-

13.- 28.-

14.- 29.-

15.- 30.-

Espacio reservado para el equipo calificador.

A B C D E

A B C D E A B C D E

A B C D E A B C D E

A B C D E A B C D E

A B C D E A B C D E

A B C D E A B C D E

A B C D E A B C D E

A B C D E A B C D E

A B C D E A B C D E

A B C D E A B C D E

A B C D E A B C D E

A B C D E A B C D E

A B C D E A B C D E

A B C D E A B C D E

A B C D E A B C D E

A B C D E

PUNTUACIÓN INCORRECTAS EN BLANCO (2) CORRECTAS (5)

Page 49: COMPENDIUM OMEM - Toomates

REAL SOCIEDAD MATEMÁTICA ESPAÑOLA

L OLIMPIADA MATEMÁTICA ESPAÑOLA

Comunidad de Madrid

FASE CERO: viernes 29 de noviembre de 2013

En la hoja de respuestas, escribe la letra de la opción que creas correcta

Cada respuesta correcta te aportará 5 puntos; cada respuesta en blanco 1 punto, y cada respuesta errónea,

0 puntos.

No está permitido el uso de calculadoras, instrumentos de medida o de cualquier aparato electrónico.

TIEMPO: 3 horas.

1. ¿Cuál es la cifra de las unidades de 32013?

A) 3 B) 9 C) 7 D) 1 E) 6

2. El robot OMEMAN se mueve sobre la recta real empezando en el 0. Da un paso a la derecha llegando al 1, dos pasos a la izquierda hasta el -1, tres pasos a la derecha hasta el 2, cuatro pasos a la izquierda hasta el -2 y así sucesivamente, alternando derecha e izquierda y un paso más en cada movimiento. ¿A qué número llegará después de 2012 giros?

A) 1007 B) -1006 C) 27 D) 11 E) 0

3. El dibujo muestra un hexágono regular dentro de un rectángulo. ¿Cuál es la suma de los cuatro ángulos marcados?

A) 90º B) 120º C) 135º D) 150º

E) Depende de la posición del hexágono respecto del rectángulo

4. Ayer fue el cumpleaños de David y de su abuela. Hoy la edad de la abuela es par y 15 veces la de David y dentro de cuatro años será el cuadrado de la edad que tenga David. ¿Cuántos años es mayor la abuela que David?

A) 42 B) 49 C) 56 D) 60 E) 64

5. Dos lados de un triángulo miden 4 y 5 cm y el tercer lado x cm, siendo x un número entero. ¿Cuántos valores diferentes puede ser x?

A) 4 B) 5 C) 6 D) 7 E) 8

6. Un triángulo tiene dos lados iguales de longitud 5. ¿Qué longitud debe tener el tercer lado para que sea máxima el área de dicho triángulo?

A) 5 B) 6 C) 25 D) 8 E) 35

7. En la figura se observa un hexágono regular ABCDEF y un triángulo EGD en el que G es un punto del lado AB. ¿Qué fracción del área del hexágono ocupa el triángulo?

A) 4

1 B)

15

2 C)

3

1 D)

5

2

E) 12

5

B G

C

D

F

E

A

Page 50: COMPENDIUM OMEM - Toomates

8. ¿Cuántas ecuaciones de segundo grado, de la forma x 2 + ax + b = 0, con a y b diferentes, tienen por solución a y b?

A) Infinitas B) Ninguna C) 4 D) 3 E) 1

9. El área del triángulo ABE es 5

1 del área del rectángulo ABCD y el

área del triángulo EFC es 8

1 del área de dicho rectángulo. ¿Qué

fracción de la superficie del rectángulo ocupa el triángulo sombreado AFE?

A) 40

27 B)

40

21 C)

2

1 D)

40

19 E)

60

23

10. Un estudio entre los hábitos alimenticios de los estudiantes de un centro escolar concluye que al 70% les gustan las peras, al 75% les gustan las naranjas, al 80% les gustan los plátanos y al 85% les gustan las manzanas. ¿Cuál es el menor porcentaje posible de estudiantes de ese centro a los que les gustan todas esas frutas?

A) 10% B) 15% C) 20% D) 25% E) 70%

11. Si los números x e y verifican que x + y = 20 y que 2

111

yx, ¿cuál es el valor de yxxy 22 ?

A) 80 B) 200 C) 400 D) 640 E) 800

12. En cada una de las doce casillas de la tabla de la figura hemos colocado un número de forma que la suma de los números de cada fila es la misma y también la suma de los números de cada una de las cuatro columnas es la misma. ¿Qué número representa la letra x?

A) 4 B) 5 C) 6 D) 7 E) 8

13. Sobre los lados mayores de un paralelogramo de lados a y b y un ángulo de 30º, construimos dos cuadrados, hacia fuera, como muestra la figura. ¿Cuál es la distancia entre los centros, P y Q, de los cuadrados?

A) abba 22 B) abab 222

C) abba 22 D) 122 ba

E) 4

22 b

a

14. En el triángulo de vértices A(0, 4), B(-2, 0) y C(4, 0) inscribimos un cuadrado con un lado descansando sobre el lado BC del triángulo. Uno de los vértices del cuadrado es el punto P de coordenadas:

A) (1, 3) B) (2, 2) C) (1,5; 2,5) D) (1,6; 2,4) E) (1,8; 2,2)

15. ¿Cuál es el mayor valor de a para el que la recta y = 10x + 5 es tangente a la parábola

62 axxy ?

A) 18 B) 16 C) 15 D) 14 E) 12

B

C D

E

F

A

2 4 2

3 3

6 1 x

b a Q

P 30º

Page 51: COMPENDIUM OMEM - Toomates

16. Si x e y son enteros positivos que verifican la ecuación yx 11 , ¿cuál es el máximo valor

de y

x?

A) 2 B) 4 C) 8 D) 11 E) 44

17. En la figura se observan tres circunferencias tangentes exteriores y dos rectas tangentes a las tres. Si los radios de las circunferencias son r1, r2 y r3, con r1 < r2 < r3) y la distancia entre el centro de la pequeña y el centro de la mayor es 16r1, el cociente

2

1

r

r es:

A) 2

1 B)

5

2 C)

3

1

D) 7

2 E)

4

1

18. En el triángulo isósceles ABC, con AB = AC, la bisectriz del ángulo de vértice C corta al lado AB en

un punto D. Si BC = CD, el ángulo ADC ˆ es igual a:

A) 90º B) 100º C) 108º D) 110º E) 120º

19. En una clase las chicas son más del 45% pero menos del 50%. ¿Cuál es el menor número posible de chicas en esa clase?

A) 5 B) 7 C) 8 D) 9 E) 10

20. Si 13·11·7·5·3·2! 23615n , el valor de n es:

A) 13 B) 14 C) 15 D) 16 E) 17

21. Si a + b + c = 7 y 10

7111

accbba, ¿cuál es el valor de

ac

b

cb

a

ba

c

?

A) 10

19 B)

10

17 C)

7

9 D)

2

3 E)

7

10

22. En la figura se observa un rectángulo ABCD de lados 20 y 15 y un arco de la circunferencia de centro en A y radio AC. La cuerda de dicho arco pasa por D y B. ¿Cuál es la longitud de dicha cuerda?

A) 50 B) 25·202 C) 13·372 D) 44

E) 26

23. Para cada entero positivo n ordenamos, de mayor a menor, todos sus divisores; el primero será el propio número y el último el 1. ¿Para cuántos valores de n el segundo número de esta lista será el 91?

A) Infinitos B) 6 C) 5 D) 4 E) 3

24. ¿Cuántos divisores de 4 cifras tiene el número 1022?

A) 1 B) 2 C) 5 D) 3 E) 4

20

F

E

D C

B A

15

Page 52: COMPENDIUM OMEM - Toomates

25. El cuadrado de la figura tiene lado igual a 1. ¿Cuál es el radio del círculo pequeño?

A) 12 B) 4

1 C)

4

2 D)

2

21

E) 212

26. En un cajón hay 2 pares de calcetines negros, 3 pares de grises y 4 de azules. Yo sé que hay 3 calcetines con agujeros pero no me acuerdo de qué color eran. Si están todos revueltos y los elijo al azar, ¿cuántos calcetines debo coger, como mínimo, para estar seguro de poder ponerme dos del mismo color y sin agujeros?

A) 2 B) 3 C) 6 D) 7 E) 8

27. En el rectángulo ABCD la bisectriz del ángulo  corta a la diagonal BD en el punto E. Si las distancias de E a los lados DC y BC son 1 y 8, respectivamente, ¿cuál es la longitud del lado AB de dicho rectángulo?

A) 228 B) 211 C) 10

D) 233 E) 2

211

28. ¿Cuál es la última cifra, distinta de cero, del número 53457 5·3·2 ?

A) 8 B) 6 C) 4 D) 2 E) 1

29. Se juntan tres cuadrados como se indica en la figura. ¿Cuánto mide el ángulo x?

A) 30º B) 40º C) 45º

D) 50º E) 60º

30. Algunos números de tres cifras verifican que si le quitas la primera cifra o si le quitas la última, obtienes, en ambos casos, un cuadrado perfecto. Por ejemplo el número 816. ¿Cuál es la suma de todos los números de tres cifras con esta curiosa propiedad?

A) 2016 B) 2013 C) 1993 D) 1465 E) 1177

8 E

D C

B A

1

x

Page 53: COMPENDIUM OMEM - Toomates

L OLIMPIADA MATEMÁTICA ESPAÑOLA

FASE CERO-COMUNIDAD DE MADRID

Primera sesión, viernes 29 de noviembre de 2013

Hoja de respuestas

Nombre y apellidos: …………………………………………….…………………………. Tfno. ………………….

Centro ………………………………………………………………… Curso …….. Fecha de nacimiento ………….

1.- 16.-

2.- 17.-

3.- 18.-

4.- 19.-

5.- 20.-

6.- 21.-

7.- 22.-

8.- 23.-

9.- 24.-

10.- 25.-

11.- 26.-

12.- 27.-

13.- 28.-

14.- 29.-

15.- 30.-

Espacio reservado para el equipo calificador.

A B C D E A B C D E

A B C D E A B C D E

A B C D E A B C D E

A B C D E A B C D E

A B C D E A B C D E

A B C D E A B C D E

A B C D E A B C D E

A B C D E A B C D E

A B C D E A B C D E

A B C D E A B C D E

A B C D E A B C D E

A B C D E A B C D E

A B C D E A B C D E

A B C D E A B C D E

A B C D E A B C D E

PUNTUACIÓN INCORRECTAS EN BLANCO (2) CORRECTAS (5)

Page 54: COMPENDIUM OMEM - Toomates

REAL SOCIEDAD MATEMÁTICA ESPAÑOLA

LI OLIMPIADA MATEMÁTICA ESPAÑOLA

Comunidad de Madrid

FASE CERO: jueves 20 de noviembre de 2014

En la hoja de respuestas, escribe la letra de la opción que creas correcta

Cada respuesta correcta te aportará 5 puntos; cada respuesta en blanco 1 punto, y cada respuesta errónea,

0 puntos.

No está permitido el uso de calculadoras, instrumentos de medida o de cualquier aparato electrónico.

TIEMPO: 3 horas.

1. La media de los cinco enteros consecutivos a, a + 1, a + 2, a + 3, a + 4 es b. ¿Cuál es la media de los cinco enteros consecutivos b, b + 1, b + 2, b + 3, b + 4?

A) a + 3 B) a + 4 C) a + 5 D) a + 6 E) a + 7

2. Dos rectas perpendiculares se cortan en el punto A(6, 8). Si las ordenadas en el origen (corte con el eje Y) de ambas rectas suman cero, ¿cuál es el área del triángulo de vértices A y dichos cortes con el eje Y?

A) 45 B) 48 C) 54 D) 60 E) 72

3. Si A > B > 0 y A es el x % mayor que B, x es igual a

A) B

BA ·100 B)

B

BA ·100 C)

A

BA ·100 D)

A

BA ·100 E)

B

A·100

4. Lanzamos al aire cuatro dados de seis caras. ¿Cuál es la probabilidad de que al menos tres de ellos muestren la misma cara?

A) 36

1 B)

72

7 C)

9

1 D)

36

5 E)

6

1

5. ¿Cuál es la mayor potencia de 2 que divide a 10 1002 – 4 501?

A) 2 1002 B) 2 1003 C) 2 1004 D) 2 1005 E) 2 1006

6. ¿Para cuántos enteros x se verifica que el número x 4 – 51x 2 + 50 es negativo?

A) 8 B) 10 C) 12 D) 14 E) 16

7. Los enteros positivos a y b verifican que las rectas y = ax + 5, y = 3x + b cortan al eje de abscisas en el mismo punto. ¿Cuál es la suma de todas las posibles abscisas de este punto de intersección?

A) – 20 B) – 18 C) – 15 D) – 12 E) – 8

8. En el interior de un cuadrado de lado 32 dibujamos dos triángulos equiláteros

como muestra la figura. ¿Cuál es el área del rombo intersección de los dos triángulos?

A) 3

34 B) 3 C) 122 D) 1238 E)

2

3

Page 55: COMPENDIUM OMEM - Toomates

9. ¿Cuántos enteros positivos x ≤ 100 verifican que x 2 + x 3 es un cuadrado perfecto?

A) 6 B) 7 C) 8 D) 9 E) 10

10. Dividimos un círculo de radio 12 en dos sectores, el menor de 120º, utilizando cada uno de ellos para formar la cara lateral de un cono. ¿Cuál es el cociente entre los volúmenes del cono pequeño y del mayor?

A) 8

3 B)

4

3 C)

10

10 D)

6

5 E)

5

5

11. En el rectángulo ABCD, con AB = 6 y AD = 30, sea G el punto medio de AD. Prolongamos desde B el lado AB hasta el punto E, con BE = 2, y sea F el punto de intersección de ED con BC. ¿Cuál es el área del cuadrilátero BFDG?

A) 2

183 B) 67 C)

2

135 D) 68 E)

2

137

12. Si la nota de cada uno de los chicos de una clase hubiera sido 4 puntos más, la media de toda la clase habría subido 1 punto. ¿Qué porcentaje de chicas había en la clase?

A) 40% B) 50% C) 75% D) 80% E) 60%

13. Esteban y Joaquín comienzan simultáneamente a caminar desde dos puntos distintos A y B, en sentido opuesto, por un mismo sendero y se cruzan al cabo de 3 horas. Esteban llega al punto desde el que salió Joaquín 2 horas y media antes de que Joaquín llegue al punto desde el que salió Esteban. ¿Cuántas horas duró la excursión de Joaquín?

A) 6 B) 6,5 C) 7 D) 7,5 E) 8,5

14. Programamos un ordenador para que transmita una cierta sucesión de cinco dígitos, ceros y unos, cinco veces. Una de las cinco veces lo hizo correctamente, otra vez tuvo un error, otra vez dos, otra vez tres y la otra vez cuatro errores. Las cinco respuestas son las que se proponen a continuación. ¿Cuál fue la correcta?

A) 00001 B) 00100 C) 01100 D) 10010 E) 10011

15. ¿Cuál es la longitud del lado AB del triángulo ABC sabiendo que AC = 3, AD = 3, BD = 8 y CD = 1?

A) 8 B) 8,75 C) 9

D) 9,25 E) 9,5

16. ¿Cuál es el mayor entero k para el que 85! es divisible entre 42 k?

A) 2 B) 13 C) 41 D) 81 E) 135

17. En un concurso de televisión hay un premio guardado en una de cinco cajas A, B, C, D y E con la misma probabilidad en cada una. El concursante elige la caja A. El realizador del concurso, que sabe en qué caja está el premio, abre dos de las otras cuatro, la B y la C, en las no está el premio. ¿Cuál es la probabilidad de que el premio esté en la caja D?

A) 5

1 B)

3

1 C)

5

2 D)

5

3 E)

3

2

18. Considera el conjunto de todas las soluciones enteras (x, y) de la ecuación x 2 = y 4 + 671. ¿Cuál es la suma de todos los valores, y, del conjunto de soluciones?

A) 0 B) 5 C) 36 D) 41 E) 1296

D C B

A

Page 56: COMPENDIUM OMEM - Toomates

19. ¿Cuál es el mínimo valor de la expresión Ryxxyxy ,142242222 ?

A) 3 B) 223 C) 5 D) 225 E) 252

20. El dominio de la función

xxf

16

116

4

14

2

1 lglglglglg)( es un intervalo de longitud n

m con m y n

primos entre sí. ¿Cuál es el valor de m + n?

A) 19 B) 31 C) 271 D) 319 E) 511

21. Si P(x) es un polinomio de tercer grado con P(0) = k, P(1) = 2k y P(– 1) = 3k, ¿cuál es el valor de P(2) + P(– 2 )?

A) 0 B) k C) 6k D) 7k E) 14k

22. En el interior del cuadrado ABCD de área 196 hay dos cuadrados que se solapan como muestra la figura. Si el área del mayor de los dos es el cuádruple de la del menor y el área de la región común a ambos es 1, ¿cuál es el área de la región sombreada?

A) 64 B) 72 C) 76 D) 80 E) 84

23. En el dibujo se observan dos semicírculos iguales y tres cuadrados. ¿Cuál es el cociente entre las áreas de las regiones sombreadas?

A) 2

1 B)

3

2 C)

4

3 D)

5

4 E)

6

5

24. El mayor número primo conocido, formado solamente por ceros y unos es el número

1

9

11010 640641

. ¿Cuántas cifras tiene?

A) 640 B) 641 C) 1280 D) 1281 E) 640·641

25. Si x, y, z son enteros positivos menores que 10 tales que 5210100 zyxzyx , ¿Cuál es

el valor de 222 zyx ?

A) 21 B) 23 C) 29 D) 33 E) 37

26. ¿Para cuántos enteros positivos n, resulta que 3 n + 81 es un cuadrado perfecto?

A) 0 B) 1 C) 2 D) 3 E) 4

27. ¿Para cuántos enteros positivos n, se verifica que el resto de la división de 1 000 063 entre n es 63?

A) 29 B) 37 C) 39 D) 49 E) 79

28. ¿Cuál es el resto de la división de 122)( 24950199200 xxxxxxxP entre )2)(1( xx ?

A) 2x – 1 B) 7 C) 2x + 3 D) 1 E) 6x – 5

D C

B A

Page 57: COMPENDIUM OMEM - Toomates

29. En la figura que observas, E es el centro de la circunferencia que pasa por A, B y D. El centro de la circunferencia que pasa por E y C es D. Si el ángulo en B es de 63º, el valor del ángulo en C es:

A) 18º B) 20º C) 22º D) 24º E) 26º

30. Si x 3, definimos xxxf 2332 lglglglg)( . ¿Cuál de las siguientes afirmaciones es verdadera?

A) 30)( xxf B) 30)( xxf C) 30)( xxf

D) 0)( xf para un único valor de x 3 E) 0)( xf para más de un valor de x 3

63º E

D

C B

A

Page 58: COMPENDIUM OMEM - Toomates

LI OLIMPIADA MATEMÁTICA ESPAÑOLA

FASE CERO-COMUNIDAD DE MADRID

Primera sesión, jueves 20 de noviembre de 2014

Hoja de respuestas

Nombre y apellidos: …………………………………………….…………………………. Tfno. ………………….

Centro ………………………………………………………………… Curso …….. Fecha de nacimiento ………….

1.- 16.-

2.- 17.-

3.- 18.-

4.- 19.-

5.- 20.-

6.- 21.-

7.- 22.-

8.- 23.-

9.- 24.-

10.- 25.-

11.- 26.-

12.- 27.-

13.- 28.-

14.- 29.-

15.- 30.-

Espacio reservado para el equipo calificador.

A B C D E A B C D E

A B C D E A B C D E

A B C D E A B C D E

A B C D E A B C D E

A B C D E A B C D E

A B C D E A B C D E

A B C D E A B C D E

A B C D E A B C D E

A B C D E A B C D E

A B C D E A B C D E

A B C D E A B C D E

A B C D E A B C D E

A B C D E A B C D E

A B C D E A B C D E

A B C D E A B C D E

PUNTUACIÓN INCORRECTAS EN BLANCO (1) CORRECTAS (5)

Page 59: COMPENDIUM OMEM - Toomates

LI OME – Comunidad de Madrid Segunda prueba. Jueves 18 de diciembre de 2014.

1. Al quitar un número de una lista de diez enteros consecutivos resulta que la suma de los nueve restantes es 2014. ¿Qué número hemos quitado?

2. En una reunión hay varias personas. Se incorpora Alicia y la media de la edad aumenta en 4 años.

Posteriormente se incorpora Beatriz, que es gemela de Alicia, y la media de edad vuelve a aumentar, pero en este caso solo en 3 años. ¿Cuántas personas había en la reunión antes de entrar Alicia?

3. Sobre los lados de un triángulo rectángulo, de catetos uno doble que el otro,

dibujamos cuadrados hacia fuera, como se muestra en la figura. El polígono obtenido lo inscribimos en un rectángulo como puede observarse en la citada figura. ¿Cuál es el cociente entre el área del polígono rayado y el área del rectángulo en el que está inscrito?

4. La gráfica de 4004022 =++ xxyy divide al plano en varias regiones una de las

cuales está acotada. Calcula el área de esa región. 5. En el triángulo ABC, de área 48, P es el punto medio de la mediana AM

y N el punto medio del lado AB. ¿Cuál es el área del triángulo MDP? 6. Una caja contiene varias bolas, todas iguales. El cociente entre el volumen ocupado por las bolas y

el volumen de la caja no ocupado por las bolas es k1 , con k entero mayor que 1.

Sacamos de la caja un número primo de bolas y ahora el cociente entre el volumen ocupado por las

restantes y el volumen de la caja no ocupado por las bolas es 2

1k

. ¿Cuántas bolas había en la caja

al principio? 7. En el interior del cuadrado ABCD, de lado 1, dibujamos dos rectángulos

iguales, AEFD y GHIJ. ¿Cuánto mide el segmento AE? 8. Uno de los lados de un triángulo mide 10 cm y la mediana que llega a ese lado, que mide 9 cm, es

perpendicular a una segunda mediana del triángulo. Calcula la longitud de la tercera mediana del triángulo.

9. En el cuadrado ABCD de lado 1 se inscribe un cuadrado de lado PQ = y,

como muestra la figura, y en uno de los triángulos rectángulos determinado por los dos cuadrados se inscribe otro cuadrado de lado x. Al moverse el punto P sobre el lado AB cambian los valores de x e y. Determina x e y para que x 2 + y 2 sea mínimo. ¿Cuánto vale ese mínimo?

10. En el triángulo ABC cuyos ángulos verifican BCA ˆº90ˆˆ <<< se trazan las bisectrices exteriores de

los ángulos A y B . Los segmentos de estas bisectrices, cada uno hasta la prolongación del lado opuesto, miden lo mismo que el lado AB. Calcula la medida del ángulo A .

N

M

D

C

B A

P

J

H

G

F

E

D C

B A

I

y x

Q

P

D C

B A

Page 60: COMPENDIUM OMEM - Toomates

52 a

54 a

52 a

a 2a

52 a 5 a

51 OME – FASE LOCAL (MADRID) SEGUNDA PRUEBA 1. Llamando a , 1+a , …, 9+a a los enteros, tenemos que ( ) ( ) ( ) 20149...1 =+−+++++ kaaaa

siendo 90 ≤≤ k , entero. Así pues 2014459 =−+ ka , ka += 19699 , 2=k , de donde 219=a y el número que hemos quitado es el 221.

2. Siendo n el número inicial de personas y x la media de sus edades, tenemos que ( ) ( )41 +⋅+=+ xnaxn

donde a es la edad de Alicia. También ( ) ( )722 +⋅+=+ xnaxn . De estas ecuaciones obtenemos

⎩⎨⎧

++=++=

1427244

xnaxna

, con lo que restando a la segunda el doble de la primera, sale 6=n

3. Llamando a y 2a a los catetos del triángulo dado, la semejanza de

triángulos que aparecen en la figura nos permite nombrar las diversas

longitudes como lo hemos hecho. Así pues, el área del polígono rayado

es 22222 1145 aaaaa =+++ y el área del rectángulo exterior es

599

565

545 22 ⋅=⎟⎟⎠

⎞⎜⎜⎝

⎛+⎟⎟⎠

⎞⎜⎜⎝

⎛+ aa por lo que el cociente pedido es

=

59911

95

4. Si 0≥x , la función es 4004022 =++ xxyy , es decir, ( )2024002 +−=− yxy que es equivalente a

( ) ( ) 022020 =+−⋅+ xyy , que responde a las rectas ⎩⎨⎧

+−=−=

20220

xyy

.

Si 0<x , tendríamos 4004022 =−+ xxyy ; ( )2024002 −−=− yxy ; ( )( ) 022020 =++− xy y que

da lugar a las rectas ⎩⎨⎧

−−==

20220

xyy

.

Así pues, la gráfica de nuestra función es la representada al lado y la

región acotada que determina es el paralelogramo de base 20 y altura

40 cuya área es 800

5. Al ser P y N puntos medios de los segmentos AM y AB respectivamente, PMNB es un trapecio en el que la base PN es la mitad de la MB. Por otra parte, ( ) 24=AMB por lo que ( ) 6=APN con lo que ( ) 18=PMNB . Finalmente, la semejanza de los triángulos PND y MBD y el hecho de que ( ) ( )PNMPNB = nos permite llamar x e y a las áreas que hemos llamado, de donde

⎩⎨⎧

=+=+6

1852yx

yx, con lo que 4=x

A B

C

M

N

P D x

x y 4y

Page 61: COMPENDIUM OMEM - Toomates

6. Definidas las variables de la siguiente forma. Volumen de la caja: V; número original de bolas: n; volumen de cada bola bV ; número de bolas que sacamos p, siendo p primo.

knVVVn

b

b 1=−⋅

; ( )( ) 2

1kV pnV

V pn

b

b =−−

−. Escribamos n en términos de p y k:

⎪⎩

⎪⎨⎧

+−=−

−=

bbbb

bb

pVnVVpVknVk

nVVknV22

. Restando y dividiendo por bV , tenemos ppknkkn −=+− 22 ;

( ) ( )( )1

11 2

2

2

−+=

−+=

kkkp

kkkpn . Así pues, k divide a ( )12 +kp , pero k y 12 +k son primos entre sí,

luego k divide a p y como 1>k , sigue que pk = . Tenemos, pues, 1

21112

−++=

−+=

pp

ppn .

Al ser n entero, 1−p divide a 2, luego 2=p ó 3. En cualquier caso, 5=n

7. Sea IJAEx == y IJC ˆ=α , x

CI sen =α ; IBCI senx −==⋅ 1α , como

1== ADIH , sigue que IB osc =α , por lo que αα senx cos1−=⋅ ,

αα

sen x cos1−= . Por otra parte, 1=++ HBEHAE , es decir:

1cos =+⋅+ αα sen xx , de donde sustituyendo x tenemos una ecuación en α sen ,

α osc : 1coscos1cos1 =+−+− αααα

αα sen

sen

sen ; es decir, ( ) 1cos1cos1 =++− αα

αα sen

sen ;

12 =α sen ,entonces º30=α ; por tanto, 23=α osc , =

−=

21

231

x 32 −

8. 10=BC , 9=AD . Como G es el baricentro, 6=AG y 3=GD y al ser 5=CD y el triángulo CGD rectángulo, es 4=CG . Nos piden BE. En el

triángulo rectángulo AGC, de catetos 6 y 4, 132=AC . Finalmente, como EG es mediana a la hipotenusa AC del triángulo rectángulo AGC, su longitud es la mitad de la hipotenusa, o sea, 13 y 132=GB , así pues 133=BE

9. Los triángulos rectángulos PQB y PRS son semejantes por lo que

PBPB

PBxx

−=

−− 11; xPBPB =− 2 . Por otra parte, ( ) 222 1 yPBPB =−+ , o sea:

122 22 +−= PBPBy , es decir, xy 212 −= , con lo que debemos hacer mínimo

xx 212 −+ . Como 2PBPBx −= y 10 ≤≤ PB , llamando tPB = , es 2ttx −=

por lo que ⎥⎦⎤

⎢⎣⎡∈

41,0x . Así pues, ( )22 121 −=−+ xxx alcanza el mínimo si

41=x y vale

169

10. Nos dicen que los triángulos APB y ABQ son isósceles con BQABAP == por lo que podemos llamar α y β como

en la figura. αβ 22ˆ2 ⋅=⋅= QBD . Así, en el triángulo

APB , α8º180ˆ −=A , por tanto, º902

8º180 =+− αα ;

º12=α

B A

C I

J D

E

F

G

H x

x

1

B

A

C D

E

G

Q R

S x

y

B A

C D

P

P

D

β

Q

B

A

C β

α

α

Page 62: COMPENDIUM OMEM - Toomates

REAL SOCIEDAD MATEMÁTICA ESPAÑOLA

LII OLIMPIADA MATEMÁTICA ESPAÑOLA

Comunidad de Madrid

FASE CERO: viernes 27 de noviembre de 2015

En la hoja de respuestas, escribe la letra de la opción que creas correcta.

Cada respuesta correcta te aportará 5 puntos, cada respuesta en blanco 1 punto y cada respuesta errónea 0

puntos.

No está permitido el uso de calculadoras, instrumentos de medida o de cualquier aparato electrónico.

TIEMPO: 3 horas.

1. ¿Cuál de los siguientes números es el mayor?

A) 44 · 777 B) 55 · 666 C) 77 · 444 D) 88 · 333 E) 99 · 222

2. Javier tarda 16 minutos en llegar desde casa al Instituto. Sus pasos son de 75 cm cada uno y da 90 pasos por minuto. Su hermano Santiago va por el mismo camino, da 100 pasos por minuto pero cada paso es de solo 60 cm. ¿Cuántos minutos tarda Santiago en llegar al Instituto?

A) 14 B) 16 C) 18 D) 20 E) 22

3. Al simplificar

2

6

4

2727

obtenemos:

A) 4

3 B)

2

3 C)

4

33 D)

2

3 E)

2

33

4. Si 2a

b y 3

b

c, ¿cuál es el cociente

cb

ba

?

A) 3

1 B)

8

3 C)

5

3 D)

3

2 E)

4

3

5. Si p, q y r son primos diferentes, ¿cuál de los siguientes números es el menor cubo perfecto que

tiene a 42·· rqpn como uno de sus divisores?

A) 88·8 ·· rqp B) 322·· rqp C) 322·2 ·· rqp D) 63·3 ·· rqp E) 33·3 ···4 rqp

6. Si 278 m , ¿cuál es el valor de 4m?

A) 3 B) 4 C) 9 D) 13,5 E) No existe ese m

7. En el interior de un cuadrado dibujamos cuatro triángulos rectángulos de hipotenusa el cuádruple de la altura sobre ella, como se muestra en la figura. ¿Cuál es el cociente entre la longitud del lado del cuadrado sombreado y el lado del cuadrado inicial?

A) 2

1 B)

2

2 C)

4

3

D) 2

3 E)

4

15

Page 63: COMPENDIUM OMEM - Toomates

8. ¿Cuál es el área de un sector circular cuyo perímetro coincide con la longitud de la circunferencia a la que pertenece si el área del círculo correspondiente es 1?

A)

1 B)

1 C)

360

D)

3

1 E)

2

1

9. En la caja A hay a monedas de 1 € y en la B, b monedas también de 1 €. Los números a y b son impares y a > b. ¿Cuál es el menor número de monedas que hay que pasar de la caja A a la B para que en la caja B haya más monedas que en A?

A) 2

2ab B)

2

2 ba C)

2

2 ab D)

2

2 ba E)

2

2 ba

10. Ali y Bea juegan un cierto número de partidas de un determinado juego. Por cada partida jugada (no hay empates) el ganador recibe 2 puntos y el perdedor recibe 1 punto. Al final, Ali ganó 4 partidas y Bea obtuvo 10 puntos. ¿Cuántas partidas jugaron?

A) 5 B) 6 C) 7 D) 8 E) No es posible determinarlo

11. Después de ir paseando durante 8 km a una velocidad de 4 km/h, Isa empieza a correr a una velocidad de 8 km/h. ¿Cuántos minutos debe estar corriendo para que la velocidad media a lo largo de todo el recorrido sea de 5 km/h?

A) 15 B) 20 C) 30 D) 35 E) 40

12. Si la media de dos números positivos es el 30 % menor que el mayor, ¿qué porcentaje será mayor que el menor?

A) 75% B) 70% C) 30% D) 25% E) 20%

13. En el cuadrado de la figura 1, E y F son los puntos medios de los lados a los que pertenecen. ¿Qué fracción del área del cuadrado representa el pentágono GECFH?

A) 3

1 B)

5

2 C)

7

3 D)

12

5 E)

15

4

Figura 1. Figura 2.

14. En el interior de un cuadrado de 24 cm de lado dibujamos cinco rectángulos iguales, como muestra la figura 2. ¿Cuál es el área, en cm2, de cada uno de ellos?

A) 12 B) 16 C) 18 D) 24 E) 32

15. El volumen de un paralelepípedo recto de 32 cm2 de área total es 8 cm3. Si las tres dimensiones están en progresión geométrica, la suma, en cm, de las longitudes de todas las aristas es:

A) 28 B) 32 C) 36 D) 40 E) 44

16. Si lanzamos un dado normal seis veces consecutivas, ¿cuál es la probabilidad de obtener un número mayor o igual que 5 al menos cinco veces?

A) 729

13 B)

729

12 C)

729

2 D)

729

3 E) Nada de lo anterior

H

G

F

E

D

C B

A

Page 64: COMPENDIUM OMEM - Toomates

17. ¿Cuántos números podemos escribir como suma de dos enteros positivos distintos, si cada uno de ellos debe ser menor o igual que 100?

A) 196 B) 197 C)198 D)199 E) 200

18. ¿Cuántos números de cuatro cifras tienen al menos una cifra repetida?

A) 62·72 B) 52·72 C) 52·82 D) 42·82 E) 42·92

19. Dos coches, A y B, recorren la misma distancia. El coche A recorre la mitad de la distancia a k km/h y la otra mitad a v km/h. El coche B va la mitad del tiempo a k km/h y la otra mitad a v km/h. Si la velocidad media de A ha sido x km/h y la de B y km/h, entonces:

A) x y B) y x C) x = y D) x < y E) x > y

20. ¿Cuántas soluciones tiene la ecuación 312 xx ?

A) 0 B) 1 C) 2 D) 3 E) 4

21. En el lado BC del triángulo ABC marcamos un punto D tal que º60ˆˆ BADDAC . Si AC = 3 y AB = 6,

la longitud de AD es:

A) 2 B) 2,5 C) 3 D) 3,5 E) 4

22. Si A, B y C no están alineados, ¿cuál es el valor mínimo del área del triángulo ABC si A(0, 0), B(36, 15) y las coordenadas de C son números enteros?

A) 2

1 B) 1 C)

2

3 D)

2

13 E) No existe tal mínimo

23. Si 10 yxx y 12 yyx , x + y es igual a:

A) – 2 B) 2 C) 5

18 D)

3

22 E) 22

24. Si tg y tg son las soluciones de la ecuación 02 qpxx y tg

1 y

tg

1 son las soluciones

de la ecuación 02 sxrx , r·s es necesariamente igual a:

A) qp· B) qp·

1 C)

2q

p D)

2p

q E)

q

p

25. En la sucesión 1, 3, 2, -1, -3,… en la que cada término, después de los dos primeros, es igual al

anterior menos el anterior al anterior, es decir, 21 nnn aaa para n > 2. ¿Cuál es la suma de los

100 primeros términos de esta sucesión?

A) 5 B) 4 C) 2 D) 1 E) -1

26. En un rectángulo de lados 6 y 11, trazamos las bisectrices de cada uno de los ángulos en los extremos de uno de los lados de longitud 11. Dichas bisectrices dividen al otro lado de longitud 11 en tres partes. Las longitudes de estas partes son:

A) 1, 9, 1 B) 6, 1, 6 C) 3, 5, 3 D) 4, 3, 4 E) 5, 1, 5

27. Si p es primo y las dos raíces de la ecuación 04442 ppxx son enteros, entonces:

A) 1 < p 11 B) 11 < p 21 C) 21 < p 31 D) 31 < p 41 E) 41 < p 51

Page 65: COMPENDIUM OMEM - Toomates

28. En el trapecio rectángulo PQRS trazamos las diagonales, siendo 5 y 10 las áreas de dos de los triángulos que determinan, como se muestra en la figura. ¿Cuál es el área del trapecio?

A) 60 B) 45 C) 40 D) 35 E) 30

29. María escribió varios enteros positivos distintos, ninguno mayor que 100, tales que su producto no era divisible por 18. ¿Cuántos escribió como mucho?

A) 5 B) 17 C) 68 D) 69 E) 90

30. ¿Cuál es el valor de la suma 1024log1023log...3log2log1log 22222 ?

A) 8192 B) 8204 C) 9218 D) !1024log2 E) Nada de lo anterior

(Recuerda. [x] representa la parte entera de x).

10

R Q

S P 5

Page 66: COMPENDIUM OMEM - Toomates

LII OLIMPIADA MATEMÁTICA ESPAÑOLA

FASE CERO-COMUNIDAD DE MADRID

Primera sesión, viernes 27 de noviembre de 2015

Hoja de respuestas

Nombre y apellidos: …………………………………………….…………………………. Tfno. ………………….

Centro ………………………………………………………………… Curso …….. Fecha de nacimiento ………….

1.- 16.-

2.- 17.-

3.- 18.-

4.- 19.-

5.- 20.-

6.- 21.-

7.- 22.-

8.- 23.-

9.- 24.-

10.- 25.-

11.- 26.-

12.- 27.-

13.- 28.-

14.- 29.-

15.- 30.-

Espacio reservado para el equipo calificador.

A B C D E A B C D E

A B C D E A B C D E

A B C D E A B C D E

A B C D E A B C D E

A B C D E A B C D E

A B C D E A B C D E

A B C D E A B C D E

A B C D E A B C D E

A B C D E A B C D E

A B C D E A B C D E

A B C D E A B C D E

A B C D E A B C D E

A B C D E A B C D E

A B C D E A B C D E

A B C D E A B C D E

PUNTUACIÓN INCORRECTAS EN BLANCO (1) CORRECTAS (5)

Page 67: COMPENDIUM OMEM - Toomates

REAL SOCIEDAD MATEMÁTICA ESPAÑOLA

LIII OLIMPIADA MATEMÁTICA ESPAÑOLA

Comunidad de Madrid

FASE CERO: viernes 25 de noviembre de 2016

En la hoja de respuestas, escribe la letra de la opción que creas correcta.

Cada respuesta correcta te aportará 5 puntos, cada respuesta en blanco 1 punto y cada respuesta errónea 0

puntos.

No está permitido el uso de calculadoras, instrumentos de medida o de cualquier aparato electrónico.

TIEMPO: 3 horas.

1. Si el perímetro del triángulo equilátero PQR es 48 cm, ¿cuál es, en cm, el perímetro del paralelogramo PSTU?

A) 26 B) 28 C) 30 D) 32

E) 34

2. La media, mediana y moda del conjunto de siete números 60, 100, x, 40, 50, 200 y 90 son todas iguales a x. ¿Cuál es el valor de x?

A) 50 B) 60 C) 75 D) 90 E) 100

3. En el dibujo que observas, el perímetro del cuadrado ABCD es 120 y el perímetro del triángulo AED es 2x. ¿Cuál de las siguientes expresiones corresponde al perímetro del pentágono ABCDE?

A) 120 + 2x B) 40 + 2x C) 60 + 2x D) 90 + 2x

E) 30 + 2x

4. Las dimensiones de una caja de base rectangular vienen expresadas por números enteros y están en la proporción 1 : 3 : 4. ¿Cuál de los siguientes números puede corresponder al volumen de la caja?

A) 18 B) 56 C) 72 D) 96 E) 144

5. En cierto campamento de verano cada uno de los 100 estudiantes que hay sabe cantar, bailar o hacer teatro pero ninguno sabe hacer las tres cosas. Hay 42 que no saben cantar, 65 que no saben bailar y 29 que no saben hacer teatro. ¿Cuántos estudiantes saben hacer dos de estas tres cosas?

A) 16 B) 25 C) 36 D) 49 E) 64

6. En el interior de un cuadrado de lado 1 hemos dibujado cinco cuadrados iguales como muestra la figura. El punto medio de cada uno de los lados del cuadrado central coincide con uno de los vértices de cada uno de los otros cuatro cuadrados. Si la longitud del lado de cada uno de estos cinco cuadrados

es b

a 2 con a y b enteros positivos, a + b es igual a:

A) 7 B) 8 C) 9 D) 10 E) 11

E

D C

B A

U

R Q

P

T

S

Page 68: COMPENDIUM OMEM - Toomates

7. ¿Qué porcentaje de números de cuatro cifras diferentes, todas impares, son divisibles entre 3?

A) 80% B) 25% C) 33% D) 40% E) 75%

8. El número 3333 podemos escribirlo como la suma de 33 impares consecutivos. El mayor de todos ellos es:

A) 3332 + 32 B) 3331 + 32 C) 3332 – 32 D) 3333 – 32 E) 3332

9. ¿Cuál es el resultado de la siguiente operación?

123456785 · 123456782 – 123456783 · 123456784

(Como ves, se trata de una resta y productos de números de nueve cifras)

A) – 2 B) – 1 C) 0 D) 2 E) Nada de lo anterior

10. En el paralelogramo ABCD de la figura, se verifica que 2

3

EB

AE.

¿Cuál es el cociente entre el área del cuadrilátero AECD y el área del paralelogramo ABCD?

A) 2

1 B)

3

2 C)

4

3 D)

5

4 E)

6

5

11. La diferencia entre una fracción positiva y su inversa es 20

9. ¿Cuál es la suma de dicha fracción y

su inversa?

A) 40

41 B)

9

20 C)

16

25 D)

20

41 E) 5

12. La base (lado desigual) del triángulo isósceles de la figura tiene 12 cm de longitud. Trazando tres rectas perpendiculares a la base, dividimos dicho triángulo en cuatro partes de igual área. ¿Cuál es la longitud del segmento x?

A) 23 B) 4 C) 4,5 D) 5 E) 33

13. Una recta que pasa por el origen de coordenadas corta a las rectas x = 1 e 13

3 xy

formando un triángulo equilátero. ¿Cuál es el perímetro de este triángulo?

A) 62 B) 322 C) 6 D) 323 E) 3

36

14. Los puntos de corte de la parábola aaxxy 222 con el eje de abscisas tienen coordenadas

enteras. ¿Cuál es la suma de todos los valores posibles de a?

A) 7 B) 8 C) 16 D) 17 E) 18

15. Ali, Bea y Celia escogen al azar un número entre los diez primeros enteros positivos, siendo diferentes todos los escogidos.¿Cuál es la probabilidad de que el número de Ali sea múltiplo del de Bea y el de Bea sea múltiplo del de Celia?

A) 64

1 B)

72

1 C)

80

1 D)

90

1 E)

200

3

16. De cuántas formas podemos escribir 345 como suma de una lista creciente de dos o más enteros positivos consecutivos?

A) 1 B) 3 C) 5 D) 6 E) 7

E B

D C

A

x

Page 69: COMPENDIUM OMEM - Toomates

17. En el interior del rectángulo grande dibujamos otros dos cuyos lados son paralelos a los del primero, como muestra la figura. Si las áreas de las tres regiones, A, B y C, están en progresión aritmética, la altura del rectángulo pequeño es 1 cm y la anchura de las regiones B y C también es de 1 cm, ¿cuánto mide la base, x, del rectángulo pequeño?

A) 1 B) 2 C) 4 D) 6 E) 8

18. Los vértices de un cuadrilátero son los puntos P(a, b), Q(b, a), R(–a, –b) y S(–b, –a), siendo a y b enteros y a > b > 0. Si el área de dicho cuadrilátero es 16, ¿cuál es el valor de a + b?

A) 4 B) 5 C) 6 D) 12 E) 13

19. ¿Cuál es la suma de las soluciones positivas de la ecuación 7218 222 xxxx ?

A) 5 B) 7 C) 8 D) 9 E) 18

20. Si x es cualquier número real, el menor valor que toma la expresión 342 xx es:

A) –1 B) 0 C) 1 D) 3 E) –3

21. La gráfica de la parábola cbxaxy 2 es la que observas, cuyo vértice está en

el eje de ordenadas. ¿Qué afirmación de las siguientes tiene que ser verdadera?

A) a + b + c = 0 B) a + b – c < 0 C) a – b + c < 0 D) a + b + c < 0

E) No hay información suficiente.

22. Si )1)(2(...)2014()2()2015()2()2016()2()( 201420152016 xxxxxxxxxp la

suma de los coeficientes del polinomio p(x) es:

A) 1008 B) 2016 C) 2027090 D) 0 E) 1

23. En un cuadrante de una circunferencia inscribimos otra circunferencia, como muestra la figura 1. ¿Cuál es el cociente entre el área del círculo pequeño y el área del cuadrante?

A) 3

2 B)

5

4 C) 323 D)

3

6 E) 2234

24. En el triángulo PQT de la figura 2, PQ = 10 cm, QT = 5 cm y el ángulo PQT = 60º. Los puntos Y, W y V son los puntos de tangencia de la circunferencia de centro O con las rectas que determinan los lados del triángulo. ¿Cuál es, en cm, el radio de dicha circunferencia?

A) 3235 B)

2

335 C)

2

135 D)

2

35 E)

6

325

25. Seleccionamos al azar tres enteros diferentes entre 1 y 2016, ambos inclusive. Si llamamos p a la probabilidad de que el producto de los tres sea impar, entonces:

A) 8

1p B)

8

1p C)

3

1

8

1 p D)

3

1p E)

3

1p

x 1

B

1

C

A

V

W

Y

T

Q P

O

Figura 1 Figura 2

Page 70: COMPENDIUM OMEM - Toomates

26. ¿Cuál es el área encerrada por la gráfica de la curva yxyx 22 ?

A) 2 B) 2 C) 22 D) 22 E) 22

27. La gráfica de 11 22 yxyyxx está formada por:

A) Dos rectas paralelas B) Dos rectas que se cortan C) Una recta y una parábola

D) Tres rectas concurrentes en un punto E) Tres rectas no concurrentes en un punto

28. Los tres vértices del triángulo ABC están en la gráfica de la parábola y = x2. El vértice A es el origen de coordenadas y el lado BC es paralelo al eje de abscisas. Si el área del triángulo es 64, ¿cuál es la longitud del lado BC?

A) 4 B) 6 C) 8 D) 10 E) 16

29. En el triángulo ABC de la figura de lados AB = 6, BC = 7 y CA = 8, las

bisectrices AD y BE de los ángulos A y B respectivamente, se cortan

en el punto F. ¿Cuál es el cociente FD

AF?

A) 2

3 B)

3

5 C) 2 D)

3

7 E)

2

5

30. En el triángulo PQR, el ángulo R es el doble del ángulo P , PR = 5 y QR = 4. ¿Cuál es la longitud del lado PQ?

A) 102 B) 6 C) 7 D) 72 E) 25

B A

E

C

F D

Page 71: COMPENDIUM OMEM - Toomates

LIII OLIMPIADA MATEMÁTICA ESPAÑOLA

FASE CERO-COMUNIDAD DE MADRID

Primera sesión, viernes 25 de noviembre de 2016

Hoja de respuestas

Nombre y apellidos: …………………………………………….…………………………. Tfno. ………………….

Centro ………………………………………………………………… Curso …….. Fecha de nacimiento ………….

1.- 16.-

2.- 17.-

3.- 18.-

4.- 19.-

5.- 20.-

6.- 21.-

7.- 22.-

8.- 23.-

9.- 24.-

10.- 25.-

11.- 26.-

12.- 27.-

13.- 28.-

14.- 29.-

15.- 30.-

Espacio reservado para el equipo calificador.

A B C D E A B C D E

A B C D E A B C D E

A B C D E A B C D E

A B C D E A B C D E

A B C D E A B C D E

A B C D E A B C D E

A B C D E A B C D E

A B C D E A B C D E

A B C D E A B C D E

A B C D E A B C D E

A B C D E A B C D E

A B C D E A B C D E

A B C D E A B C D E

A B C D E A B C D E

A B C D E A B C D E

PUNTUACIÓN INCORRECTAS EN BLANCO (1) CORRECTAS (5)

PREGUNTA ANULADA

Page 72: COMPENDIUM OMEM - Toomates

A

B

P

Q

LIII OME - SEGUNDA PRUEBA FASE LOCAL, COMUNIDAD DE MADRID

21 de diciembre de 2016

1. El producto de dos números del conjunto { }26 , ... ,3 ,2 ,1 es igual a la suma de los restantes.

Encuentra dichos números. 2. Un “capicúa” es un número que se lee igual de izquierda a derecha que de derecha a

izquierda. (Por ejemplo, 323 ó 19591). ¿Cuál es la diferencia entre el mayor y el menor capicúa, si ambos son de cinco cifras y múltiplos de 45?

3. Dividimos el trapecio de la figura en cuatro triángulos trazando las

diagonales. Si X e Y son las áreas de los triángulos sombreados, obtén en función de X e Y el área del trapecio.

4. Considera las ecuaciones de la forma 02 =++ cbxax en las que a, b y c son números primos de una sola cifra. ¿En cuántas de estas ecuaciones hay al menos una solución entera?

5. En una bolsa hay bolas rojas y bolas azules, en total menos de 2016. Sabemos que la

probabilidad de que al coger dos bolas (sin reemplazamiento) sean ambas del mismo color,

es 12

. ¿Cuál es el máximo número de bolas rojas que puede haber en la bolsa?

6. Los puntos de corte de la parábola aaxxy 22 +−= con el eje de abscisas, tienen

coordenadas enteras. ¿Cuál es la suma de todos los valores posibles de a?

7. El dibujo muestra dos circunferencias y dos rectas tangentes a ambas

Siendo A, B, P y Q los puntos de tangencia. Si la longitud del segmento PQ es 14 y la del AB es 16, calcula el producto de los radios de las circunferencias.

8. En una circunferencia de centro O y diámetro AB marcamos un punto C (distinto de A y B)

desde el que trazamos la perpendicular al diámetro AB, al que corta en el punto D. Si M es un punto de la cuerda BC tal que º90=OMB

⌢ y OMDB ⋅= 3 , calcula el ángulo CBA ˆ .

9. Hay un único triángulo ABC para el que 14=AC , 54A cos = y el radio del círculo inscrito es

4. Calcula el área de dicho triángulo.

10. Sean x, y, z números reales tales que: 2

93421 zyxzyx ++=−+−+− . Determinar el

valor de zyx 32 ++ .

X

Y

Page 73: COMPENDIUM OMEM - Toomates

REAL SOCIEDAD MATEMÁTICA ESPAÑOLA

LIV OLIMPIADA MATEMÁTICA ESPAÑOLA

Comunidad de Madrid

FASE CERO: viernes 24 de noviembre de 2017

En la hoja de respuestas, escribe la letra de la opción que creas correcta.

Cada respuesta correcta te aportará 5 puntos, cada respuesta en blanco 1 punto y cada respuesta errónea 0

puntos.

No está permitido el uso de calculadoras, instrumentos de medida o de cualquier aparato electrónico.

TIEMPO: 3 horas.

1. Colocamos cifras en los huecos del número 2, una en cada hueco, para formar un número de

tres cifras. ¿De cuántas formas podemos hacerlo para que el número obtenido sea mayor que 217?

A) 81 B) 82 C) 83 D) 92 E) 93

2. En la siguiente gráfica el punto P está en el eje OY, Q es el (4, 0) y la recta PQ pasa por el punto R(2, 4). ¿Cuál es el área del triángulo OPQ?

A) 8 B) 12 C) 32 D) 24 E) 16

3. En el triángulo de la figura, el punto M es el punto medio

del lado AB, º30ˆ BMC y º15ˆ BAC . ¿Cuánto mide el

ángulo ABC ˆ ?

A) 75º B) 65º C) 60º

D) 80º E) 85º

4. Jorge tiene 144 cubitos idénticos de 1 cm de arista. Utiliza todos para construir un prisma rectangular cuya base tiene un perímetro de 20 cm, pero hay distintas posibilidades. ¿Cuál es la suma de todas las posibles alturas del prisma?

A) 31 B) 25 C) 15 D) 22 E) 16

5. Irene es más baja que Jorge, Francisco es más alto que Gustavo, Jorge es más alto que Francisco y Herminia es más baja que Gustavo. ¿Quién es el más alto?

A) Francisco B) Gustavo C) Herminia D) Irene E) Jorge

6. El cociente entre la longitud del lado menor de un rectángulo y la longitud del lado mayor es igual al cociente del lado mayor y la diagonal. ¿Cuál es el cuadrado del cociente entre la longitud del lado menor y la diagonal?

A) 2

13 B)

2

1 C)

2

53 D)

2

2 E)

2

16

7. En el triángulo ABC, AB = 6, AC = 8 y BC = 10. Si D es el punto medio del lado BC, ¿cuál es la suma de los radios de las circunferencias inscritas en los triángulos ADB y ADC?

A) 51 B) 4

11 C) 53 D)

6

17 E) 3

R(2, 4)

P

Q O

M

C

B A

Page 74: COMPENDIUM OMEM - Toomates

8. El cuadrado PQRS, de lado 42, está dividido en cuatro rectángulos del mismo perímetro, tal y como muestra la figura. ¿Cuál es el área del rectángulo sombreado?

A) 252 B) 432 C) 441 D) 490

E) 540

9. Las longitudes de los lados de un triángulo obtusángulo son:10, 17 y x. Si x es un número entero, ¿cuál es la suma de los posibles valores de x?

A) 161 B) 148 C) 63 D) 323 E) 224

10. La región del espacio formada por los puntos que distan 3 unidades del segmento AB tiene por

volumen 216. ¿Cuál es la longitud de dicho segmento?

A) 6 B) 12 C) 18 D) 20 E) 24

11. Conduciendo a velocidad constante, Alberto tarda 3 horas en ir desde su casa a casa de sus padres. Un día empezó a conducir a su velocidad habitual pero, después de llevar la tercera parte del camino, empezó a llover y redujo su velocidad en 20 km/h, tardando en total 276 minutos. ¿Qué distancia hay entre la casa de Alberto y la de sus padres?

A) 132 km B) 135 km C) 138 km D) 141 km E) 144 km

12. Isa tiene 30 varillas, de longitudes enteras y diferentes, entre 1 y 30 cm. Toma tres de ellas, de longitudes 3 cm, 7 cm y 15 cm y las coloca encima de una mesa. Debe elegir una cuarta para formar con las cuatro un cuadrilátero. ¿Cuántas de las 27 restantes puede elegir?

A) 16 B) 17 C) 18 D) 19 E) 20

13. En un triángulo rectángulo, de lados 3, 4 y 5, inscribimos de dos formas diferentes dos cuadrados. El primero, de lado x, tiene un vértice que coincide con el vértice del triángulo correspondiente al ángulo recto. El segundo, de lado y, tiene dos vértices consecutivos en la hipotenusa. ¿Cuál es el

valor de y

x?

A) 13

12 B)

37

35 C) 1 D)

35

37 E)

12

13

14. En el rectángulo ABCD, AB = 3 y BC = 4. El punto E es el pie de la perpendicular desde B a la diagonal AC. ¿Cuál es el área del triángulo AED?

A) 1 B) 25

42 C)

15

28 D) 2 E)

25

54

15. Prolongamos por B el diámetro AB, de una circunferencia de radio 2, hasta un punto D de tal forma que BD = 3. Elegimos un punto E tal que ED = 5 y los segmentos ED y AD sean perpendiculares. El segmento AE corta a la circunferencia en el punto C, entre A y E. ¿Cuál es el área del triángulo ABC?

A) 37

100 B)

39

140 C)

39

145 D)

37

140 E)

31

120

16. ¿Cuántos enteros positivos menores o iguales que 2017, escritos en la notación habitual, llevan la cifra cero?

A) 469 B) 471 C) 475 D) 478 E) 481

Page 75: COMPENDIUM OMEM - Toomates

17. Si a y b son números reales positivos y las raíces de las ecuaciones 022 baxx y

022 abxx son todas reales, el menor valor posible a + b es:

A) 2 B) 3 C) 4 D) 5 E) 6

18. La ecuación ax 12 tiene exactamente tres raíces reales. ¿Cuál es el valor de a?

A) 0 B) 1 C) 3

7 D) 3 E) 3

19. En la figura adjunta se observan dos cuadrados: el ABCD, de área

S, y el PQRT, de área S’. Si los puntos de intersección dividen a

los lados del cuadrado ABCD en tres partes iguales, ¿cuál de las siguientes afirmaciones es verdadera?

A) S = S’ B) 2S = 3S’ C) 5S = 6S’ D) 8S = 9S’

E) Nada de lo anterior

20. El número x es positivo y 71

2

2 x

x . ¿Cuánto vale 5

5 1

xx ?

A) 55 B) 63 C) 322 D) 123 E) 749

21. Si x e y son números positivos que verifican [x]·x = 36, [y]·y = 71, x + y es igual a:

A) 8

107 B)

8

119 C)

9

125 D)

6

107 E)

7

101

Recuerda: [a] “parte entera de a” es el mayor entero menor o igual que a.

22. Pedro elige tres enteros positivos a, b y c. Quino determina el valor de c

ba y obtiene como

respuesta 101. Rosa calcula bc

a y obtiene 68. Sara determina el valor de

c

ba y obtiene como

resultado k. ¿Cuál es el valor de k?

A) 13 B) 168 C) 152 D) 12 E) 169

23. Amalia tiene una moneda defectuosa en la que la probabilidad de obtener cara al realizar un

lanzamiento es de 3

1 y Bruno tiene otra en la que la probabilidad de obtener cara es de

5

2. Tira

cada uno, alternativamente, su moneda empezando Amalia y gana el primero que obtenga cara.

Si q

p, irreducible, es la probabilidad de que gane Amalia, q – p es igual a:

A) 1 B) 2 C) 3 D) 4 E) 5

24. En el triángulo equilátero ABC prolongamos desde B el lado AB hasta el punto B’ de tal manera que BB’ = 3AB. Análogamente en los otros dos lados: CC’ = 3BC y AA’ = 3CA. ¿Cuál es el cociente entre el área del triángulo A’B’C’ y el área del triángulo ABC?

A) 9 B) 16 C) 25 D) 36 E) 37

25. ¿Cuántos triángulos hay que tengan los vértices en los puntos (i, j) donde i y j son enteros del 1 al 5, ambos inclusive?

A) 2128 B) 2148 C) 2154 D) 2160 E) 2300

B

P

T

R

Q

D C

A

Page 76: COMPENDIUM OMEM - Toomates

26. Sea S el conjunto de puntos (x, y) del plano tales que dos de los tres números, 3, (x + 2), (y + 4)

son iguales y el tercero es menor o igual que esos otros dos. ¿Cuál de las siguientes es una

correcta descripción de S ?

A) S es un punto B) S es un par de rectas que se cortan C) S es un triángulo

D) S es: tres rectas que se cortan dos a dos en tres puntos diferentes

E) S es: tres semirrectas con un punto común

27. Los lados AB y AC del triángulo equilátero ABC son tangentes a una circunferencia en los puntos B y C, respectivamente. ¿Qué fracción del área de dicho triángulo cae fuera de la circunferencia?

A) 3

1

27

34 B)

82

3 C)

2

1 D)

9

323 E)

27

34

3

4

28. Los tres vértices del triángulo equilátero están en la hipérbola x·y = 1, siendo uno de los vértices de la hipérbola el baricentro del triángulo. ¿Cuál es el área de dicho triángulo?

A) 48 B) 60 C) 108 D) 120 E) 13

29. En un cuadrado de lado a trazamos dos cuadrantes de circunferencia, como muestra la figura, con centros en A y B. ¿Cuál es el área de la parte sombreada?

A) 2

4

3

3a

B) 2

6

1a C) 2

6a

D) 2

6

13a

E) Faltan datos

30. Si tgxxcossenx)x(f 32 (con x en radianes), ¿en qué intervalo está el menor valor positivo de

x para el que f(x) = 0?

A) (0, 1) B) (1, 2) C) (2, 3) D) (3, 4) E) (4, 5)

A

C

B

D

Page 77: COMPENDIUM OMEM - Toomates

LIV OLIMPIADA MATEMÁTICA ESPAÑOLA

FASE CERO-COMUNIDAD DE MADRID

Primera sesión, viernes 24 de noviembre de 2017

Hoja de respuestas

Nombre y apellidos: …………………………………………….…………………………. Tfno. ………………….

Centro ………………………………………………………………… Curso …….. Fecha de nacimiento ………….

1.- 16.-

2.- 17.-

3.- 18.-

4.- 19.-

5.- 20.-

6.- 21.-

7.- 22.-

8.- 23.-

9.- 24.-

10.- 25.-

11.- 26.-

12.- 27.-

13.- 28.-

14.- 29.-

15.- 30.-

Espacio reservado para el equipo calificador.

A B C D E A B C D E

A B C D E A B C D E

A B C D E A B C D E

A B C D E A B C D E

A B C D E A B C D E

A B C D E A B C D E

A B C D E A B C D E

A B C D E A B C D E

A B C D E A B C D E

A B C D E A B C D E

A B C D E A B C D E

A B C D E A B C D E

A B C D E A B C D E

A B C D E A B C D E

A B C D E A B C D E

PUNTUACIÓN INCORRECTAS EN BLANCO (1) CORRECTAS (5)

Page 78: COMPENDIUM OMEM - Toomates

LIV OLIMPIADA MATEMÁTICA ESPAÑOLA

FASE CERO-COMUNIDAD DE MADRID

Primera sesión, viernes 24 de noviembre de 2017

Hoja de respuestas

Nombre y apellidos: …………………………………………….…………………………. Tfno..………………….

Centro.………………………………………………………………… Curso.…….. Fecha de nacimiento.………….

1.- 16.-

2.- 17.-

3.- 18.-

4.- 19.-

5.- 20.-

6.- 21.-

7.- 22.-

8.- 23.-

9.- 24.-

10.- 25.-

11.- 26.-

12.- 27.-

13.- 28.-

14.- 29.-

15.- 30.-

Espacio reservado para el equipo calificador.

A B C D E A B C D E

A B C D E A B C D E

A B C D E A B C D E

A B C D E A B C D E

A B C D E A B C D E

A B C D E A B C D E

A B C D E A B C D E

A B C D E A B C D E

A B C D E A B C D E

A B C D E A B C D E

A B C D E A B C D E

A B C D E A B C D E

A B C D E A B C D E

A B C D E A B C D E

A B C D E A B C D E

PUNTUACIÓN INCORRECTAS EN BLANCO (1) CORRECTAS (5)

Page 79: COMPENDIUM OMEM - Toomates

1. Considera un entero N de cinco cifras. Forma el entero P, de seis cifras, colocando un 2 al comienzo de

N, y el entero Q, también de seis cifras, colocando un 2 al final de N. Si Q = 3P, escribe todos los valores

posibles de N.

2. En la figura adjunta se observan dos cuadrados: APQR y ASTU,

que tienen un vértice común, A. Si SR = 10 y M es el punto medio

de UP, calcula la longitud de AM.

3. Noryne tiene una lista de varios enteros consecutivos y efectúa el producto de todos ellos, obteniendo

como resultado un número N, de seis cifras, que empieza por 47 y acaba en 74. (N = 47…74).

Escribe la lista que tiene Noryne.

4. En la figura adjunta puedes ver una circunferencia tangente a cuatro arcos

iguales, siendo cada uno de ellos la cuarta parte de una circunferencia.

Si AB = 2 cm, calcula el radio de la circunferencia.

5. Si eliges cuatro dígitos diferentes, ninguno igual a cero, sabes que es posible formar 24 números de

cuatro cifras cada uno, todas distintas.

¿Cuál es el mayor divisor primo de la suma de estos 24 números?

6. En la figura puedes ver una “pirámide” de cuadrados de lado 1 con las siguientes condiciones:

▪ El número de cuadrados de la fila de abajo es impar (nueve en el

dibujo).

▪ Cada fila, salvo la de abajo, tiene dos cuadrados menos que la

que tiene debajo.

▪ Cada cuadrado se apoya en dos cuadrados de la fila que tiene

debajo.

▪ En la fila superior hay un cuadrado solo.

Si la “pirámide” tiene n filas, encuentra una relación que exprese su área A en función de su perímetro P.

7. Los habitantes del planeta EMO son rojos o verdes y tienen 2, 3 o 4 cabezas. Colocamos en fila a seis

de ellos, cada uno con una de las seis características citadas, es decir, R2, R3, R4, V2, V3, V4, de forma

que cualesquiera dos que estén juntos difieren en el color y en el número de cabezas.

¿De cuántas formas los podemos colocar en esa fila?

8. El entero positivo N tiene exactamente seis divisores. Si el producto de cinco de ellos es 648, ¿cuál es el

otro divisor de N?

9. Hugo y María (ambos con menos de 80 años pero más de 10) observan que si escriben sus edades una a

continuación de la otra (primero la de Hugo), obtienen un número de cuatro cifras que es un cuadrado

perfecto. A continuación, Hugo se da cuenta de que eso volverá a pasar dentro de 17 años. ¿Cuál es la

edad actual de Hugo?

10. En {a, b, c, d} la suma de todos los productos de dos factores es ab + ac + ad + bc + bd + cd y la

suma de todos los productos de tres factores es: abc + abd + acd + bcd.

Sea f(n) la suma de los productos de n factores de los 2017 primeros enteros positivos.

Así por ejemplo f(1) = 1 + 2 +…+ 2017. ¿Cuál es el valor de f(1) + f(2) + f(3) +…+ f(2017)?

10

M U

T

S R

Q

P

A

A B

Page 80: COMPENDIUM OMEM - Toomates

REAL SOCIEDAD MATEMÁTICA ESPAÑOLA

LV OLIMPIADA MATEMÁTICA ESPAÑOLA Comunidad de Madrid

FASE CERO: viernes 23 de noviembre de 2018

En la hoja de respuestas, escribe la letra de la opción que creas correcta.

Cada respuesta correcta te aportará 5 puntos, cada respuesta en blanco 1 punto y cada respuesta

errónea 0 puntos.

No está permitido el uso de calculadoras, instrumentos de medida o de cualquier aparato

electrónico.

TIEMPO: 3 horas.

La suma de dieciocho enteros consecutivos podría ser…

A) 1818 B) 1821 C) 1823 D) 1825 E) 1827

¿Qué área, en m2, tiene el triángulo de lados 7 m, 24 m y 25 m?

A) 300 B) 84 C) 87,5 D) 56 E) 168

¿Para cuántos enteros n se cumple que 64 < 8n < 32

10?

A) 47 B) 1 C) 15 D) 14 E) 4

En un segmento hemos marcado los puntos LVOME en ese orden. Sabiendo que: ME = VM,

LE – LO = 35 cm, LV = 2VO, LO = OM, ¿qué distancia, en cm, hay de V a M ?

A) 5 B) 10 C) 15 D) 20 E) 25

En la figura se ve un cuadrado de lado 1 y una circunferencia cuyo

diámetro está sobre uno de los lados del cuadrado y además es tangente a

una diagonal del cuadrado. ¿Cuánto mide el radio de la circunferencia?

A) 12 B) 2

2 C)

2

12 D)

2

22 E)

3

2

En este entramado queremos colocar los números desde el 0 hasta el 10 de

tal manera que las casillas de dos números consecutivos no se toquen (ni

siquiera en un vértice) ¿Cuánto suman los números que han de colocarse en

de las casillas rayadas?

A) 14 B) 15 C) 16 D) 17 E) 18

Al multiplicar un número de cinco cifras por 101 obtengo un número que acaba en …8965.

¿Cuánto suman las cuatro últimas cifras del número de partida?

A) 28 B) 12 C) 16 D) 17 E) 21

¿Cuántos números de tres cifras cumplen que son múltiplos de 12 y sus cifras suman 12?

A) 20 B) 19 C) 18 D) 17 E) 16

1

2

3

4

6

8

5

7

1

2

6

0

4

Page 81: COMPENDIUM OMEM - Toomates

Las soluciones de la inecuación x

x

x

x 3

1

1

son los valores del conjunto…

A) ,0S B)

,2

2

1,0S C) 2,1

2

1,0

S

D) ,11,S E)

,21,

2

1S

En un triángulo de lados a, b y c se cumple que abcbacba 3))(( . La medida del

ángulo opuesto al lado c es:

A) 15º B) 30º C) 45º D) 60º E) 150º

El valor de 5log

80loges:

A) 2log1

2log31

B) 2log4 C) 75log D) –3 E) 5log80log

¿De cuántas maneras podemos sentar a tres chicos y a tres chicas de forma alterna (no

puede haber dos personas de igual sexo juntas) y sin dejar huecos en una fila de diez

asientos?

A) 180 B) 360 C) 1800 D) 5! · 5! E) 240

Elegimos al azar un número a del conjunto {11, 13, 15, 17, 19} y otro número b del conjunto

{2016, 2017, 2018, 2019}. ¿Cuál es la probabilidad de que el número ab termine en 1?

A) 5

1 B)

4

1 C)

10

3 D)

20

7 E)

5

2

En una empresa radical, los sueldos semanales son proporcionales a la raíz cuadrada del

número de horas trabajadas. Una empleada ha calculado que si hubiera trabajado a horas

más, habría ganado p euros más; y si hubiese trabajado b horas más, habría ganado q euros

más (a y b diferentes). ¿Cuál es el sueldo semanal de la trabajadora en términos de a, b, p, q

A) )(2

22

ba

qp

B)

ab

qp

2

)( 2 C)

)(2

22

bqap

bqap

D)

)(2

22

aqbp

bpaq

E) ))(( qbba

Dos circunferencias, una interior y otra exterior, comparten el mismo centro. La longitud de

una cuerda de la mayor que es tangente a la circunferencia interior mide 16 cm. ¿Cuál es el

área, en cm2, de la corona circular limitada por dichas circunferencias?

A) 36 B) 46 C) 49 D) 64 E) 25

Al resolver el sistema de ecuaciones

4

10

yyxx

yyxx se obtienen dos soluciones

diferentes, ),(),( bayx y ),(),( dcyx . ¿Cuál es el valor de a + b + c + d ?

A) 3 B) 2 C) 0 D) 6 E) 10

9

10

11

12

16

15

14

13

Page 82: COMPENDIUM OMEM - Toomates

Si a, b y c son números positivos, el área del triángulo situado en el primer cuadrante y

limitado por los ejes y la recta de ecuación cbyax , es:

A) 2

ab B)

c

ab

2 C)

ab

c

2

2

D) 2

abc E)

22

2

ba

c

Una circunferencia de radio 1 está inscrita en un triángulo equilátero que a su

vez está inscrito en un rectángulo que está inscrito en una circunferencia. ¿Cuál

es el diámetro de la circunferencia mayor?

A) 21 B) 2 C) 32 D) 3 E) 123

El número N tiene 99 cifras y todas ellas son el 9, N = 99…9. ¿Cuánto suman las cifras del

número N×N ?

A) 9×99 B) 1+9×99 C) 99 D) 990 E) 900

En el rectángulo de la figura hemos marcado dos segmentos y tres

ángulos. ¿Cuál es el valor de

sentg

sen

x

yA ?

A) senA B) yA C) x

yA D)

xA

1 E) 1A

La función f cumple que 1)(2)]([ 2 xxfxf para todos los valores x de su dominio. Si

sabemos que f(x) es siempre positivo, ¿cuál es el dominio de la función f ?

A) )( fD B) ),1()( fD C) )1,1()( fD

D) ),0[)( fD

E) ),1[)( fD

En el rectángulo de la figura, de dimensiones 3 × 4, hemos trazado algunos

segmentos aprovechando vértices y puntos medios de lados. ¿Cuál es el

área del cuadrilátero sombreado?

A) 1,2 B) 1,75 C) 1 D) 1,25 E) 1,5

Solo uno de los siguientes números es un cuadrado perfecto. ¿Cuál?

A) 2

!28!27 B)

2

!29!28 C)

2

!30!29 D)

2

!31!30 E)

2

!32!31

Si en un triángulo isósceles los ángulos iguales aumentaran un 10%, el ángulo desigual

disminuiría un 8%. ¿Cuál es la diferencia entre el ángulo mayor y cualquiera de los menores?

A) 50º B) 48º C) 56º D) 60º E) 45º

Don Retorcido no se olvida de vosotros. Él es muy prudente y circula siempre a 40 km/h.

¿Cuántas horas tardará en realizar un recorrido de k km si necesita hacer p paradas de m

minutos cada una para escribir problemas?

A) 120

23 pmk

B) pmk 23 C)

12

23 pmk D)

40

pmk E)

40

40 pmk

17

18

19

20

25

24

21

23

22

99 cifras

x

y

Page 83: COMPENDIUM OMEM - Toomates

Para valores permitidos de x e y, la igualdad )(logloglog yxyx es cierta si…

A) Siempre B) 1

2

y

yx C) 1 yx D)

1

y

yx E)

1

1

y

yx

¿Qué radio tiene la circunferencia inscrita en un sector de radio r y 60º

grados de amplitud?

A) 2

r B)

4

r C)

2

3r D)

3

r E)

6

r

En un triángulo rectángulo de hipotenusa x, un cateto es triple que el otro. ¿Cuál es el área de

dicho triángulo en función de su hipotenusa?

A) 20

3 2x B)

10

3 2x C)

9

2x D)

4

2x E)

20

2x

Si A>B>0, completa esta frase: “si A es un M % mayor que B, entonces, B es un ….. %

menor que A”

A) BM

A B)

M

AB C)

A

BM D)

AB

M E)

M

1

Si formamos todas las palabras (con sentido o no) posibles bailando las letras A-D-D-I-M-R y

las ordenamos alfabéticamente, empezaríamos por ADDIMR y terminaríamos por RMIDDA.

¿Qué lugar ocuparía la palabra MADRID en esta lista?

A) 260 B) 246 C) 366 D) 250 E) 226

28

26

18

19

20

21

22

24

25

26

27

28

29

30

27

30

29

60º

r

r

Page 84: COMPENDIUM OMEM - Toomates

REAL SOCIEDAD MATEMÁTICA ESPAÑOLA

LV OLIMPIADA MATEMÁTICA ESPAÑOLA Comunidad de Madrid

FASE CERO: viernes 23 de noviembre de 2018

Problema Respuesta Problema Respuesta

1 E 16 D

2 B 17 C

3 D 18 A

4 D 19 A

5 A 20 E

6 C 21 B

7 D 22 D

8 D 23 E

9 B 24 D

10 D 25 A

11 A 26 B

12 B 27 D

13 E 28 A

14 D 29 C

15 D 30 B

Page 85: COMPENDIUM OMEM - Toomates

REAL SOCIEDAD MATEMÁTICA ESPAÑOLA

LVI OLIMPIADA MATEMÁTICA ESPAÑOLA Comunidad de Madrid

FASE CERO: sábado 26 de octubre de 2019

• En la hoja de respuestas, escribe la letra de la opción que creas correcta.

• Cada respuesta correcta te aportará 5 puntos, cada respuesta en blanco 1 punto y cada

respuesta errónea 0 puntos.

• No está permitido el uso de calculadoras, instrumentos de medida o de cualquier aparato

electrónico.

• TIEMPO: 3 horas.

Un grupo de 25 estudiantes se presenta a una prueba en la que pueden obtener una nota entera

desde 0 hasta 100. ¿Cuál puede ser la máxima diferencia entre la nota media y la mediana de

este grupo de estudiantes?

A) 48 B) 50 C) 40 D) 52 E) 60

Los números enteros positivos 30, 72, y N tienen la propiedad de que el producto de dos

cualesquiera de ellos es divisible entre el tercero. ¿Cuál es el menor valor posible de N?

A) 120 B) 180 C) 30 D) 60 E)10

He dibujado tres circunferencias (de radios 2, 3 y 10 cm) tangentes entre sí

como ves en el dibujo. ¿Cuál es el área, en cm2, del triángulo cuyos vértices

son los centros de las circunferencias?

A) 28 B) 20,5 C) 10 D) 15 E) 30

Un número n de cuatro cifras es capicúa. También es capicúa el número n + 312, que tiene

cinco cifras. ¿Cuál es la suma de las cifras de n?

A) 18 B) 14 C) 32 D) 33 E) 34

En la sucesión a1, 6, a3, …, la suma de cuatro términos consecutivos es constante, y también

es constante la diferencia –el mayor menos el menor– de dos términos consecutivos

cualesquiera. Sabiendo además que a1 < a2 = 6 < a3, ¿cuánto vale la suma de los 2019

primeros términos de la sucesión?

A) 2019 B) 2020 C) 4038 D) 12 114 E) 12 120

¿Para qué valores del número real k la ecuación 02 =++ kxkx tiene dos soluciones reales

de distinto signo?

A) 10 k B) 4k C) 0k D) 0k E) 1k

1

2

3

4

5

6

Page 86: COMPENDIUM OMEM - Toomates

P O Q

En un pentágono regular ABCDE de lado 1, las diagonales AC y BE se cortan en P. ¿Cuánto

mide el segmento PC?

A) 1 B) 2

5 C) 15 − D) ( )254 − E)

2

15 −

¿Cuántos subconjuntos A del conjunto B = {1, 2, 3, …, 19, 20} cumplen que:

[número de elementos de A] × [máximo elemento de A] = 18?

A) 380 B) 20 C) 3 D) 190 E) 19

Los dos lados opuestos de un cuadrado aumentan su longitud en un 25%. ¿En qué porcentaje

deben disminuir los otros dos lados para que el área del rectángulo resultante sea la misma

que la del cuadrado inicial?

A) 20% B) 22,5% C) 25% D) 40% E) 42%

Un polinomio )(xP cumple que 12)1( 242 +−=+ xxxP . ¿Cuál de las siguientes expresiones

es igual a )2( 2 +xP ?

A) 24 24 +− xx B) 24 2xx − C) 14 24 +− xx D)

4x E) 22 24 +− xx

En el dibujo mostramos un triángulo rectángulo de hipotenusa PQ en el

que hay inscrita una semicircunferencia de radio 12 cm y centro O. Si

PO mide 15 cm, ¿cuántos cm mide OQ?

A) 24 B) 20 C) 25 D) 22,5 E) 18

Si el número 9999=N está formado exclusivamente por 99 nueves, ¿cuánto suman las

cifras del número N 2?

A) 1782 B) 900 C) 891 D) 899 E) 990

La suma de un número más su inverso es 7. ¿Cuánto vale la suma del cubo de dicho número

más el inverso de su cubo?

A) 322 B) 700 C) 343 D) 294 E) 336

De cierta función f sabemos que 4)1( =f y que )()()()( yxfyxfyfxf −++= para todos

los reales x, y. ¿Cuál es el valor de )3(f ?

A) 194 B) 160 C) 52 D) 48 E) 62

7

8

9

10

11

12

13

14

Page 87: COMPENDIUM OMEM - Toomates

En el cuadrilátero ABCD, AB = 24, BC = 20, CD = 15, DA = 7 y la diagonal BD mide 25.

¿Cuánto mide la diagonal AC?

A) 18 B) 214 C) 20 D) 21 E) 24

Si xn −= 120 es entero, ¿cuántos valores puede tomar x?

A) 3 B) 6 C) 9 D) 10 E) 11

Si los términos 5º y 8º de una progresión geométrica son 7! y 8!, ¿cuál es el primer término de

esa progresión?

A) 60 B) 75 C) 120 D) 225 E) 315

El lado del cuadrado LUCA es 20. En el interior de LUCA tomamos un punto P de modo que

el triángulo PUC sea equilátero. Si 32 baLP −= con a y b enteros, ¿Cuál es el valor de b?

A) 350 B) 400 C) 450 D) 500 E) 625

Los números retorcidos son enteros positivos múltiplos de 4, y tales que al sumarles 1, el

nuevo número es múltiplo de 5, y al sumarles 2 lo es de 6. ¿Cuántos números retorcidos son

menores que 2019?

A) 16 B) 17 C) 18 D) 33 E) 34

En la celda central de un tablero 5 × 5 colocamos una ficha. Un movimiento de la ficha

consiste en desplazarla a una celda con la que comparta un vértice. Tras realizar al azar 12

movimientos, ¿cuál es la probabilidad de llegar a una de las esquinas del tablero?

A) 3

1 B)

25

4 C)

6

1 D)

13

1 E)

4

1

¿Para cuántos enteros n positivos se consigue que nn 182 + sea un cuadrado perfecto?

A) Uno B) Dos C) Tres D) Cuatro E) Ninguno

Pensemos ahora en números naturales con la siguiente propiedad: la suma de sus cuatro

divisores es 42. ¿Cuántos números cumplen esta propiedad?

A) Ninguno B) 1 C) 2 D) 3 E) 4

Tengo una bolsa llena de canicas idénticas. La suma de los pesos de todas las parejas posibles

es 630 gramos. La suma de los pesos de todas las ternas posibles es 4095 gramos. ¿Cuántos

gramos pesa cada canica?

A) 2 B) 3 C) 4 D) 5 E) 6

15

17

16

18

19

20

22

23

21

Page 88: COMPENDIUM OMEM - Toomates

El triángulo ABC es isósceles, con AB = AC. La bisectriz interior del ángulo ABC corta al lado

AC en el punto P. Si la circunferencia circunscrita al triángulo BPC pasa por el punto medio

de AB, el ángulo BAC mide…

A) 30º B) 45º C) 60º D) 90º E) 105º

En una urna hay tres bolas numeradas con los números 1, 2 y 3. Sacamos una al azar,

anotamos el número y la devolvemos a la urna. Volvemos a realizar este proceso dos veces

más y al finalizar sumamos los tres números obtenidos, que resulta ser 6. ¿Cuál es la

probabilidad de que haya salido la bola del número 2 en las tres ocasiones?

A) 2

1 B)

6

1 C)

7

1 D)

8

1 E)

27

1

Si 1

1)(

+=

x

xxf y 12 x , entonces )( xf − es igual a:

A) )(

1

xf B)

)(

1

xf− C) )(xf D) )(xf− E)

)(

11

xf−

Dos rectas con pendientes 2

1 y 2 se cortan en el punto )2,2(P . El área del triángulo

determinado por estas dos rectas y la que tiene de ecuación 10=+ yx es:

A) 4 B) 24 C) 6 D) 8 E) 26

Si n es un entero positivo tal que 440!)!2()!1( =+++ nnn , la suma de las cifras de n es:

A) 3 B) 5 C) 6 D) 10 E) 11

Del triángulo ABC sabemos que AB = AC y que el ángulo desigual mide 40º. Si la altura

trazada desde C corta a su base AB en el punto D con BD·BA = 32 cm, ¿qué longitud, en cm,

tiene el lado BC?

A) 8 B) 24 C) 4 D) 6 E) 10

Don Retorcido ha dibujado veinte puntos en una circunferencia, todos ellos igualmente

espaciados. Y se despide con esta pregunta: ¿cuántos puntos como mínimo tienes que elegir al

azar para asegurarte de que cuatro de ellos serán vértices de un rectángulo?

A) 4 B) 8 C) 12 D) 11 E) 16

24

25

26

27

30

28

29

Page 89: COMPENDIUM OMEM - Toomates

REAL SOCIEDAD MATEMÁTICA ESPAÑOLA

LVI OLIMPIADA MATEMÁTICA ESPAÑOLA Comunidad de Madrid

FASE CERO: sábado 26 de octubre de 2019

PROBLEMA RESPUESTA PROBLEMA RESPUESTA

1 A 16 E

2 D 17 E

3 E 18 B

4 E 19 E

5 D 20 E

6 D 21 B

7 A 22 B

8 E 23 B

9 A 24 C

10 D 25 C

11 B 26 A

12 C 27 C

13 A 28 D

14 C 29 A

15 C 30 C

Page 90: COMPENDIUM OMEM - Toomates
Page 91: COMPENDIUM OMEM - Toomates
Page 92: COMPENDIUM OMEM - Toomates
Page 93: COMPENDIUM OMEM - Toomates
Page 94: COMPENDIUM OMEM - Toomates

Madrid (Fase 1) 2020 Soluciones (Versión personal)

1

En primer lugar vemos que los triángulos AQD y BQD son semejantes (por Tales), y

puesto que ADAMBN )2/1( , se deduce que ABBQABBQ )2/1( .

Con el mismo razonamiento llegamos a BQABPA .

Los triángulos BQN , PAM y MND son congruentes.

Las diagonales de un paralelogramo se cortan en sus puntos medios, luego

962

1 ABCDMNCD , 24

4

1 MNCDMNO , y finalmente

21624964848 MNOABMNBQNPAMPQO

2

Estudiamos la última cifra de la sucesión na , para ello solo hay que calcular la última cifra de

las operaciones:

,...)2,1,9,0,7,1,4,5,7,6,9,5,2,1(

Y vemos que se van repitiendo cada 12 elementos.

3121682019

Luego 2019a acaba igual que 3a , es decir, en 5.

3

35

211

35

2 xyyx

xy

yx

yx

. Sea

35

xyk .

Tenemos

0352035235)2(35

2222

kkxxkkxxkxkx

kxy

xkykyx

Luego kkkkkk

x 352

3514)2(2 22

Pero x es un entero, luego kk 352 debe ser un cuadrado perfecto. El primer k para el que

esto sucede es 36k , y entonces:

63636363536363636353636 2 x

y por tanto

63663672 y

Page 95: COMPENDIUM OMEM - Toomates

El menor valor posible para la suma se encuentra en 30x , 42y , con suma 72 .

Fuente de la solución: Soluciones oficiales.

Observación: Mediante la desigualdad AM-GM encontramos una cota menor, pero que se

alcanza cuando yx , incumpliendo una de las condiciones del enunciado:

)(35222422

2yxxyxyyxxyyxxy

yx

70)(702

yxyxyx

Y la igualdad acontence solo cuando yx , que se puede resolver:

352)2(35 2 xxx , pero esta solución no es aceptable.

4

Este problema se resuelve fácilmente mediante una tabla de casos:

Los casos posibles son 1920 T , y los casos favorables son

1992

191818...321

F

Con lo que la probabilidad es 20

9

1920

199

T

FP

5

1

ba

bababababaabba

En donde para poder simplificar hemos tenido en cuenta que 0 ba .

Luego 12121112

2 abbaabbababa

y la igualdad se puede alcanzar, por ejemplo con 0a y 1b .

6

Observamos que si d es un divisor de n , también lo es dn/ , y que el producto de ambos es

n .

Luego del enunciado deducimos que con todos los divisores de n se pueden hacer cuatro

parejas de producto n , es decir, que el número n tiene 8 divisores. Está claro que los valores

más pequeños los encontraremos con factorizaciones con 1 o dos factores primos.

Con un factor primo: apn , con 8 divisores es

7pn , y el más pequeño es 12827 n .

Page 96: COMPENDIUM OMEM - Toomates

Con dos factores primos: baqpn , con 8 divisores tenemos 23qpn , y el valor más pequeño

es 24323 n

Entre las dos opciones posibles el menor valor es el segundo, 24n .

7

Aplicando el Teorema de la bisectriz, 229229

bBDa

b

BD

a

Pero aBC es un entero, y bAC y 229AD son coprimos, luego 229 es un divisor de

BD, es decir, kBD 229 para cierto k .

Y por tanto )1(229229229 kkDABDc

bkbkbBD

a

229

229

229

Por otro lado, )229)(1()1(2206733 bkkbkbcba

La única combinación aceptable es 2k y 444673229 bb

y en este caso 8884442 aBC

8

Primera versión.

Sea ),,,( dcba un código válido, con 60 a , 30 b , 40 c , 20 d . Vamos a

organizar todos los casos en función de las parejas ),( db , y su suma db

a) 80 badb 6 casos.

b) 71 badb 10 casos.

c) 62 badb 15 casos.

d) 53 badb 20 casos.

Page 97: COMPENDIUM OMEM - Toomates

e) 44 badb 25 casos.

f) 35 badb 29 casos.

En total hay:

21029252015252015102015106 casos.

Segunda versión. En las soluciones oficiales encontramos una versión mucho más elegante.

Sea T el conjunto de todas las combinaciones posibles ),,,( dcba con 60 a , 30 b ,

40 c , 20 d . Está claro que el número de elementos de T es 4203547 .

Podemos establecer una biyección entre el subconjunto S de T cumpliendo, además,

8 dcba con el subconjunto que no cumple esta última condición, es decir:

7 dcba , en efecto:

)2,4,3,6(),,,( dcbadcba

Se comprueba facilmente que la función está bien definida, y que

7815)(1524368 dcbadcbadcbaS

Así pues, el conjunto S tendrá 2102/420 elementos.

Page 98: COMPENDIUM OMEM - Toomates

9

Sean 12789 ,,...,,, aaaaa las cifras de n , siendo 1a las cifras de las unidades.

Puesto que n es múltiplo de 12, también lo será de 2, luego 1a debe ser par. Pero puesto que,

además, cualquier permutación de sus cifras también lo es, todas las cifras de n serán pares.

Puesto que es divisible entre 12, lo será también entre 4.

Tomamos todas las terminaciones 12 ,aa de los múltiples de 4:

00, 04, 08, 12, 16, 20, 24, 28, 32, 36, 40, 44, 48, 52, 56, 60, 64, 68, 72, 76, 80, 84, 88, 92, 96

Y descartamos las que contienen dígitos impares o cero, nos quedan:

24, 28, 44, 48, 64, 68, 84, 88

Es decir, todas acaban en 4 0 8. Como, además, cualquier permutación de n también tiene que

cumplir esta condición, deducimos que todos las cifras ia son 4 o 8.

Supongamos que n se escribe con k cuatros y k10 ochos, para cierto 100 k . Sabemos

que es divisible entre 3, luego la suma S de sus cifras también lo debe ser:

)2(47248724808804)10(84 kkkkkkkS es divisible entre 3 si y

solo si 8,5,2k .

Por último, puesto que n es múltiplo de 11, la suma alternada de sus cifras será también

múltiplo de 11 o 0.

Para 2k

Si los dos cuatros ocupan posiciones con la misma paridad:

4)44488(88844 no es aceptable

Si los dos cuatros ocupan posiciones con paridad distinta:

0)88884(88884 es aceptable

Se cumple la condición. Tengo 2555 combinaciones posibles (uno en una de las

cinco posiciones impares y el otro en una de las cinco posiciones pares)

Para 8k

Si los dos ochos ocupan posiciones con la misma paridad:

8)44444(44488 no es múltiplo de 11

Si los dos ochos ocupan posiciones con diferente paridad:

0)44448(44448 es aceptable.

De nuevo tengo 2555 combinaciones posibles.

Para 5k

20)88888(44444

12)88884(84444

4)88844(88444

4)88444(88844

12)84444(88884

20)44444(88888

Ninguna combinación es aceptable

Así pues, hay 502525 números geniales.

Fuente de esta solución: Soluciones oficiales.

Page 99: COMPENDIUM OMEM - Toomates

10

Para la resolución de este problema nos basaremos en el siguiente resultado:

Sea un triángulo ABC y sean M y N los respectivos puntos medios de los lados AB y AC.

Entonces ABCAMN 4

1

Volviendo al problema, sean N, M, X, Y, Z los respectivos puntos medios de AC, AB, BQ, RQ

y MQ. Puesto que en todo momento estamos trabajando con puntos medios, está claro que los

puntos X, Y, Z y P están alineados.

2

75

275

2150

2

300

ZQNPZQP

MQNAZQNPMQNA

4

75

164

ABCXQZ

ABCMBQ

4

225

2

75

4

75 ZQPXQZXQP

Puesto que la mediana divide todo triángulo en dos triángulos con igual área:

2

PQRYQP

Luego:

454

5

4

24

225

RBQRBQ

RBQRBQ

RBQXQY

RPQXQYYQPXQYXQP

Y finalmente:

902 RBQPQR

Nota: En las soluciones oficiales se presenta una versión alternativa.

Page 100: COMPENDIUM OMEM - Toomates

Madrid 2021 (fase 0)

1. De los siguientes números, ¿cuál puede expresarse como suma de los cuadrados de dos

enteros que sean múltiplos de 3?

(A) 633 (B) 459 (C) 549 (D) 363 (E) 495

2. Dos rectángulos idénticos de 2 × 5 se solapan como ves en el dibujo. ¿Cuál es el área de la

zona gris común a ambos rectángulos?

(A) 10/3 (B) 29/5 (C) 5/12 (D) 18/5 (E) 21/5

3. Si a, b, c, d son enteros con a + bc = 20 y – a + cd = 19, el mayor valor posible de c es:

(A) 45 (B) 33 (C) 42 (D) 39 (E) 29

4. Queremos formar listas de dos o más enteros positivos distintos ordenados de menor a mayor

que sumen 15. ¿Cuántas listas diferentes podremos formar?

(A) 19 (B) 27 (C) 29 (D) 26 (E) 24

5. ¿Cuántos puntos de la recta 3x + 5y =15 equidistan de los ejes de coordenadas?

(A) Uno (B) Dos (C) Ninguno (D) Infinitos (E) Tres

6. Esta lista de siete números [2; 5; 10; 2; 4; 2; x] tiene esta bonita propiedad: si ordenas su

media, moda y mediana de menor a mayor se forma una progresión aritmética no constante.

¿Cuál es la suma de todos los valores posibles que puede tomar x?

(A) 9 (B) 17 (C) 6 (D) 3 (E) 20

7. Definimos la función F(n) para n entero positivo de la siguiente forma: F(n) = 2n si n es par

y F(n) = 3n si n es impar. Si p es un número primo mayor que 2, ¿cuál es el valor de

F[F(p-1) - p]?

(A) 2(p - 2) (B) 3p – 2 (C) p – 2 (D) 2p – 2 (E) 3(p - 2)

8. La suma de los inversos de cuatro enteros positivos distintos es 3/4. Si tres de ellos están en

progresión geométrica de razón 2, ¿cuánto vale la suma de esos cuatro números?

(A) 57 (B) 55 (C) 59 (D) 53 (E) 51

Page 101: COMPENDIUM OMEM - Toomates

9. Sabiendo que log 6 = 0,778 y log 5 = 0,699, ¿cuántos de estos logaritmos puedes calcular sin

ayudarte de una calculadora? (log indica logaritmo en base 10)

(A) Uno (B) Tres (C) Dos (D) Todos (E) Cuatro

10. Si rellenamos la cuadrícula de forma que cada número es la media de los dos números que

hay en las casillas adyacentes, ¿qué número habría que colocar en la casilla sombreada?

(A) 15 (B) 16 (C) 18 (D) 17 (E) 19

11. Un cuadrado, con lados de longitudes enteras medidas en metros, tiene un área de S metros

cuadrados y un perímetro de P metros. De las siguientes opciones, solo una puede ser el valor

de S + P. ¿Cuál?

(A) 2021 (B) 2023 (C) 2024 (D) 2028 (E) 2020

12. A Jorge le han dado la paga para que vaya al cine y se compre un refresco. La entrada de

cine le ha costado el 10 % de la diferencia entre la paga y el precio del refresco. Y la bebida es

el 4% de la diferencia entre su paga y el precio de la entrada. ¿Qué porcentaje de estos se

aproxima mejor al porcentaje de paga que le queda a Jorge después de estos gastos?

(A) 82% (B) 84% (C) 85% (D) 87% (E) 88%

13. Omesia es un país en el que hay tres tipos de monedas de distinto valor y cada una de las

cuales vale una cantidad entera de euros. Bomesio, Comesia y Domesio tienen cada uno al

menos una moneda de cada tipo. Bomesio tiene cuatro monedas, que equivalen a 28 euros.

Comesia tiene cinco monedas que valen 21 euros y Domesio tiene solo tres monedas. ¿A

cuántos euros equivalen las monedas de Domesio?

(A) 19 (B) 20 (C) 16 (D) 18 (E) 17

14. En el triángulo PQR, PQ = PR = 40 cm y S es un punto en QR tal que PS = 25 cm. La

prolongación de PS corta a la circunferencia circunscrita a PQR en el punto T. ¿Cuánto mide,

en cm, PT?

(A) 68 (B) 64 (C) 67 (D) 65 (E) 60

Page 102: COMPENDIUM OMEM - Toomates

15. Tres fruteros desprendidos se encuentran con sus cestos llenos de manzanas y se las

regalan, unos a otros, de esta curiosa manera: al principio Antonio regala manzanas a Jose y

Teresa para que ambos dupliquen sus manzanas. Luego, Jose hace lo mismo y duplica las

manzanas de Antonio y Teresa. Y para terminar, Teresa les regala manzanas a sus dos amigos

para que dupliquen su número. Lo curioso es que, después de todo este tejemaneje, Teresa tenía

al final 36 manzanas, ¡las mismas con las que empezó! ¿Cuántas manzanas tienen entre los tres

amigos?

(A) 216 (B) 280 (C) 108 (D) 198 (E) 252

16. ¿Cuántos triángulos puede dibujar don Retorcido usando como vértices los puntos de esta

cuadrícula?

(A) 2148 (B) 2400 (C) 2100 (D) 2290 (E) 2300

17. Si el perímetro de un rectángulo es p y su diagonal es d, su área es…

(A) A (B) D (C) B (D) C (E) E

18. Las circunferencias grandes son iguales y la central pasa por los centros de las otras dos.

Tanto la circunferencia mediana como la pequeña son tangentes a las circunferencias grandes

en los puntos de contacto. Si el área de la circunferencia mediana es 36, ¿cuál es el área de la

circunferencia pequeña?

(A) 8,2 (B) 12 (C) 7,2 (D) 9 (E) 10,5

19. Luisa ha olvidado el código de cuatro dígitos de su móvil. Solo recuerda que todas las cifras

eran distintas, que estaban ordenadas de menor a mayor (como en 0479) y que ninguna era 8.

¿Cuántas combinaciones posibles hay?

(A) 240 (B) 420 (C) 126 (D) 729 (E) 504

Page 103: COMPENDIUM OMEM - Toomates

20. Dividimos una circunferencia de radio 1 en cuatro arcos iguales y “volteamos” dos de ellos

como ves. ¿Cuál es el área de la figura sombreada?

(A) C (B) E (C) B (D) D (E) A

Page 104: COMPENDIUM OMEM - Toomates

2021 OME Fase Local de Madrid, Segunda prueba (“Fase l”)

Bloque 1. Problemas de respuesta numérica. Debes escribir un número entero.

1

La circunferencia inscrita en un triángulo ABC es tangente a los lados AB, BC y CA en

puntos P, Q y R respectivamente. Se sabe que el radio de la circunferencia es 21, que

BQ = 27 y que AR = 23. Determina el perímetro de ABC.

2

¿Cuántos números de tres cifras diferentes, ninguna de ellas nula, son múltiplos de 3?

3

¿Cuántos tríos de números enteros positivos, A, B, C, cumplen a la vez estas tres

condiciones?

mcm (A, B) = 72 , mcm (A, C) = 600 , mcm (B, C) = 900

4

Las soluciones de la ecuación 023 txsxrx son ba , cb , ac , siendo a, b

y c las soluciones de la ecuación 01143 23 xxx . Determina t.

5

Diremos que un entero positivo n es k-especial si tiene exactamente k divisores

positivos y además es múltiplo de k. Por ejemplo, el número 18 es 6-especial, pues es

múltiplo de 6 y tiene 6 divisores positivos. ¿Cuánto vale la suma de todos los números

20-especiales menores que 2020?

Bloque 2. Problemas de respuesta corta. La respuesta no debe ocupar más de una cara de un A4.

6

Encuentra todas las ternas pnm ,, de números enteros positivos con p primo que

verifican 2144 mpn

7

En la figura se ve un cuadrado ABCD, un cuarto de circunferencia Γ1 con centro en el

vértice A y tangente a los lados BC y CD, y una semicircunferencia Γ2 con centro en el

lado CD y tangente al lado BC y a Γ2. T es el punto de tangencia entre Γ1 y Γ2. Si M es

el punto medio de AD, demuestra que C, T y M están alineados.

Page 105: COMPENDIUM OMEM - Toomates

8

Jaime ha señalado sobre una circunferencia los 21 vértices de un polígono regular de 21

lados. Ahora quiere colorear algunos de ellos de rojo de manera que los segmentos que

determinan estos puntos rojos sean todos de distinta longitud. ¿Cuántos puntos rojos a lo

sumo podrá colorear?

9

En un castillo encantado hay n habitaciones idénticas, numeradas de 1 a n, cada una de

las cuales tiene k puertas alineadas. En la habitación j, 11 nj hay una única puerta

que te lleva a la habitación j+1 y en la habitación n hay una única puerta que te saca del

castillo. Todas las demás puestas te llevan de vuelta a la habitación 1. Cuando pasas una

puerta y llegas a una habitación, no puedes saber en qué habitación estás entrando ni

qué puertas has usado con anterioridad. Empiezas en la habitación 1 y conoces los

valores de n y k. Determina para qué valores de n y k existe una estrategia que te

garantiza que podrás salir del castillo, explica en qué consiste la estrategia y demuestra

que funciona.

10

Consideramos la función de variable real xxf 3)( . Demuestra que la ecuación

xfxf 20202020

tiene una única solución y hállala.

Page 106: COMPENDIUM OMEM - Toomates

Soluciones.

1

Puesto que los segmentos tangentes con un punto común tienen la misma longitud, sabemos

que 23 APAR , 27 BQBP . Sea CQCRx .

Entonces, el semiperímetro es xxs 502723

Por un lado, aplicando GA/11.4.8,

srABC

Por otro lado, aplicando la fórmula de Heron (GA/10.5.11):

3250)27()23()50( xxxsxsssABC

Obteniendo la ecuación:

2/2456)50(21

50506)50(21

3250)50(21

2

22

xxx

xxxx

xxx

Descartando el valor negativo, llegamos a la única solución aceptable:

34524510022

24550 sps

2

Buscamos tres números diferentes 9,,0 cba , tales que cba sea divisible entre tres.

Podemos suponer que cba y multiplicar todos los resultados obtenidos por 63 P .

Puesto que 24789 cba , podemos ordenar por casos:

a) 3 cba . Ningún caso.

b) 6 cba . Un caso:

1+2+3

c) 9 cba . Tres casos:

1+2+6, 1+3+5, 2+3+4

d) 12 cba . Siete casos:

1+2+9, 1+3+8, 1+4+7, 1+5+6, 2+3+7, 2+4+6, 3+4+5

e) 15 cba . Ocho casos:

1+5+9, 1+6+8, 2+4+9, 2+5+8, 2+6+7, 3+4+8, 3+5+7, 4+5+6

f) 18 cba . Siete casos:

1+8+9, 2+7+9, 3+6+9, 3+7+8, 4+5+9, 4+6+8, 5+6+7

g) 21 cba . Tres casos:

4+8+9, 5+7+9, 6+7+8

h) 24 cba . Un caso:

7+8+9

Sumamos 30 casos, multiplicando por sus 6 permutaciones respectivas, hacen un total de 180

casos.

Observación: En las soluciones oficiales aparecen desarrollos alternativos.

Page 107: COMPENDIUM OMEM - Toomates

3

a) 23 3272),( BAmcm

b) 23 532600),( CAmcm

c) 222 532900),( CBmcm

Aplicando la factorización en factores primos y que el mcm de dos números toma los factores

repetidos y no repetidos con el exponente máximo, vemos que A, B y C tienen todos factorización

en primos 2, 3, 5.

Con respecto al factor 5:

Por a) vemos que ni A ni B son divisibles entre 5, luego por b) o c) se deduce que C tiene factor 25 .

Un solo caso.

Con respecto al factor 3:

Por a) vemos que A o B tienen factor 23 . Pero si lo tuviera A, entonces aparecería en b), luego

deducimos que B tiene factor 23 y que A y C tienen factores 13 o 03 , y que al menos uno de

los dos tiene que tener factor 13 . Son tres casos posibles:

Con respecto al factor 2:

Por b) vemos que A o C, al menos uno de los dos, tiene que tener factor 32 . Pero si lo tuviera

C, aparecería en c), luego deducimos que A tiene factor 32 , y que B tiene factor 22 , 12 o 02 , lo

mismo que C, pero al menos uno de los dos tiene que tener factor 22 . Son cinco casos posibles:

En total son 15 casos posibles.

Page 108: COMPENDIUM OMEM - Toomates

4

Aplicamos las fórmulas de Vieta para ecuaciones de tercer grado (ver PA/3.1):

Queremos determinar ))()(( accbbat sabiendo que

abc

cabcab

cba

abc

cabcab

cba

11

4

3

11

4

)(3

Luego:

23

23111211)3(4))(())()((

t

abccbacabcabaccbbat

5

Sea n un número 20-especial.

La factorización en factores primos de n debe ser de la forma kc

k

ccba pppn ...52 21

21 para ciertos 2a y 1b porque n es múltiplo de 5220 2 .

Supongamos en primer lugar que n no tiene más factor primo que 2 y 5, es decir, que es de la

forma ban 52 .

Ya sabemos (ver AR/19.4) que el número de divisores de n viene dado por la fórmula

)1)(1()( ban

y por tanto las posibilidades válidas son:

20202560521,921,101

20202000523,441,51

20205000524,351,41

5220)1)(1()(

9

34

43

2

nbaba

nbaba

nbaba

ban

Solo hay una posibilidad aceptable: 34 52 n .

Supongamos que existe un tercer factor primo, diferente de 2 y 5: cba pn 52 .

Entonces solo se cumple para la configuración siguiente:

25

202080521,1,421,21,51

5220)1)(1)(1()(

4

2

p

ppncbacba

cban

Y por tanto:

2352,2152,1952,1752,1352,1152,752,352 44444444 n

Con más de tres factores primos es imposible que tenga 20 divisores, luego hemos acabado el

recuento, y la suma es:

9440

935529352522319171311735252

2352195217521352115275235252

24434434

444444434

Page 109: COMPENDIUM OMEM - Toomates

6

1212144144 22 mmmpmp nn

Es decir:

nbapm

pm

b

a

,

12

12

Imponiendo nba ,0 llegamos a

npm

mm

12

11112

Que no es aceptable.

Imponiendo 0, bna llegamos a

2,525513112

12 2

npp

mm

pm n

n

Obteniendo la terna solución 5,2,13

Supongamos que 0, ba

Si a la primera ecuación le restamos la segunda obtenemos: ba pp 24

De lo que deducimos que forzosamente ba , y en este caso:

124323 babba pppp

Con 2p , una combinación aceptable es

2012221285,3,212232 55233 mmnabp

Obteniendo la solución 2,8,20

Con 3p , una combinación aceptable es

1512331243,1,313332 23213 mmnabp

Obteniendo la solución 3,4,15

Con ninguna otra combinación obtenemos soluciones aceptables, así que las soluciones son

tres: 5,2,13 , 2,8,20 , 3,4,15 .

Page 110: COMPENDIUM OMEM - Toomates

7

En el cuadrado ABCD trazamos la circunferencia Γ1 con centro A. Sea Γ2 la circunferencia

tangente a Γ1 con centro O en CD y que pasa por C.

Trazamos la recta TC. Sea TE su punto de corte con Γ1, y sea M su punto de corte con AD.

Queremos demostrar que M es el punto medio del segmento AD.

El triángulo OTC es isósceles, pues OTOC , luego ETAOTCOCT .

Pero el triángulo EAT también es isósceles, pues ATAE , luego

OCTETATEA

Luego OCTTEA y por tanto OCEA // .

Pero la única recta paralela a OC que pasa por A es AB, luego E pertenece a AB, y EB es un

diámetro de la circunferencia Γ1.

Por el teorema del punto medio, puesto que A es el punto medio de EB, luego la perpendicular

AD por A (que es paralela a BC) cortará EC por su punto medio, es decir, M es el punto medio

de EC, o dicho de otra forma, el punto medio de AD pertenece a la recta TC.

Nota: En las soluciones oficiales podemos encontrar una versión bastante elegante mediante

homotecias: Sea E el simétrico de B respecto de AD, y sea O el centro de la circunferencia Γ2.

La homotecia que transforma Γ2 en Γ1 tiene centro T y envía C a E, pues las homotecias

conservan el paralelismo y OC// AB. Así pues, C, T y E están alineados. Puesto que A es el

punto medio de EB, está claro que el punto medio M de AD pertenece a EC.

Page 111: COMPENDIUM OMEM - Toomates

8

Marcando como 0 uno de los vértices, y observando la simetría de la figura que obtenemos:

Diez vértices a la derecha numerados 1, 2, 3, 4, 5, 6, 7, 8, 9, 10 y diez vértices a la izquierda,

numerados 11, 12, 13, 14, 15, 16, 17, 18, 19, 20, vemos que la configuración más grande de

vértices que determinan segmentos con longitud diferente se obtiene con 10,6,3,1,0

9

Existe una estrategia para cualquier valor de n y de k. Numeramos las puertas en orden, de

izquierda a derecha como k...,,2,1 . Luego escapar del castillo equivale a encontrar la

combinación correcta

nppp ,...,, 21 con kpi 1

Es decir, debemos probar todas las variaciones posibles de los k números tomados en cadenas

de longitud n, pero garantizando que empezamos en la habitación 1, puesto que al probar una

combinación, podemos salir del castillo pero también podemos acabar en cualquiera de sus

habitaciones.

Por tanto, nuestro problema se reduce a encontrar una combinación que garantize que nos

situamos en la habitación 1, saliendo de una habitación cualquiera, una especie de “reset”.

La clave está en ver que si kq 11 es la puerta que lleva de la habitación 1 a la habitación 2,

entonces, tomando cualquier 1qp , la cadena de longitud n

ppp ,...,,

Nos deja seguro en la habitación 1 independientemente de donde estemos (o bien nos saca del

castillo).

En efecto, aplicando la cadena anterior, independientemente de donde estemos, o bien salimos

del castillo, o bien nos situamos en la habitación 1, de donde ya no saldremos, pues 1qp .

Asi pues, supongamos que queremos probar la combinación nppp ,...,, 21 .

Tomamos un 1pp y aplicamos previamente la combinación ppp ,...,, .

Si 1qp , entonces la combinación ppp ,...,, nos dejará seguro en la habitación 1 y ahora

probamos la combinación nppp ,...,, 21 , que nos puede o no sacar del castillo.

Si 1qp , puesto que entonces 11 qp , da igual porque la combinación a probar no sería la

correcta.

Fuente de esta solución: Soluciones oficiales.

Page 112: COMPENDIUM OMEM - Toomates

10

Sea 20202020 33)2020( txxfxt

Luego

2020

20203

320203

32020320203202020202020

t

txxfxf

t

tx

Consideremos la función xxF x 3)( .

Queremos resolver )2020(32020)( 2020 FtF .

La función F es estrictamente creciente, pues es la suma de dos funciones estrictamente

crecientes, luego necesariamente 2020t , y por tanto: 20202020 320203 tx

Fuente de esta solución: Soluciones oficiales.